[obm-l] Re: [obm-l] conesul

2002-07-31 Thread yurigomes


 Fala Carlos Tenta mostrar que o médio de DE, o incentro do ABC e o
ponto T são colineares.
 Falow!!
-- Mensagem original --

>Alguem fez a 2 de geom. da conesul desse ano? empaquei nela...
>Obrigado,
>Carlos
>

[]'s, Yuri
ICQ: 64992515


--
Use o melhor sistema de busca da Internet
Radar UOL - http://www.radaruol.com.br



=
Instruções para entrar na lista, sair da lista e usar a lista em
http://www.mat.puc-rio.br/~nicolau/olimp/obm-l.html
O administrador desta lista é <[EMAIL PROTECTED]>
=



[obm-l] Re: [obm-l] E o Nivel Tres,ninguem faz nada??????

2002-10-26 Thread yurigomes
 Não. A condição de ser conjunto não permite repetição de elementos. Com
repetição estamos falando de um MULTISET ( ou multiconjunto ).

-- Mensagem original --

> --- Eduardo Casagrande Stabel <[EMAIL PROTECTED]>
>escreveu: > Olá.
>> 
>> O Pessoal da Lista envelheceu junto com a Lista, por
>> isso só se ouve sobre a Universitária.
>> 
>> Eu encontrei uma solução muito simples para essa
>> questão.
>> 
>> Seja P > 1 + 2 + 3 + ... + 2002 um número primo.
>> O conjunto A = { P, 2P, 3P, ..., 2002P } satisfaz o
>> enunciado pois se x é a soma de alguns elementos de
>> A então temos P <= x <= P + 2P + 3P + ... + 2002P <
>> P.P = P^2. Portanto P <= x < P^2 e x é múltiplo de
>> P, logo não é uma potência perfeita pois P^2 precisa
>> dividir x.
>> 
>> Abraço,
>> Eduardo.
>
>Olá,
>Eu dei exatamente este exemplo na minha solução. Outro
>
>exemplo que eu citei foi o trivial A =
>{2003,2003,2003,
>...,2003} (ou outro primo qualquer maior que 2002).
>Será que aceitam que existam elementos iguais no
>conjunto?
>
>[]'s,
>Murilo Vasconcelos,
>Maceió, AL
>
>___
>Yahoo! GeoCities
>Tudo para criar o seu site: ferramentas fáceis de usar, espaço de sobra
e
>acessórios.
>http://br.geocities.yahoo.com/
>=
>Instruções para entrar na lista, sair da lista e usar a lista em
>http://www.mat.puc-rio.br/~nicolau/olimp/obm-l.html
>O administrador desta lista é <[EMAIL PROTECTED]>
>=
>

[]'s, Yuri
ICQ: 64992515


--
Use o melhor sistema de busca da Internet
Radar UOL - http://www.radaruol.com.br



=
Instruções para entrar na lista, sair da lista e usar a lista em
http://www.mat.puc-rio.br/~nicolau/olimp/obm-l.html
O administrador desta lista é <[EMAIL PROTECTED]>
=



[obm-l] Re: [obm-l] [IME96] Provar que eh periódica:

2002-10-27 Thread yurigomes
  Tente primeiro calcular f(x+2a) em função de f(x). Daí, vc chegará em
f(x+2a)= 1/2 + sqr( 1/4 - f(x) + f(x)^2)= 
 = 1/2 + sqr((1/2 - f(x))^2). Mas sabemos que f(x) é maior ou igual a 1/2,
pois f(x)= 1/2 + sqr(algo), donde f(x)-1/2 é maior ou igual a zero. Logo,
concluímos que 
   f(x+2a)= 1/2 + (f(x) - 1/2)= f(x), pra todo x

-- Mensagem original --

>
>
>  Provar que a equação (anexada) eh periódica:
>
>  Alguém pode me ajudar com essa?
>
>  Fui!
>
>
>### Igor GomeZZ 
> UIN: 29249895
> Vitória, Espírito Santo, Brasil
> Criação: 27/10/2002 (01:54)
>
>Pare para pensar:
>
>Só quando a última árvore for
>derrubada, o último peixe for
>morto e o último rio for poluído é
>que o homem perceberá que não pode
>comer dinheiro. (Greenpeace)
>
>


[]'s, Yuri
ICQ: 64992515


--
Use o melhor sistema de busca da Internet
Radar UOL - http://www.radaruol.com.br



=
Instruções para entrar na lista, sair da lista e usar a lista em
http://www.mat.puc-rio.br/~nicolau/olimp/obm-l.html
O administrador desta lista é <[EMAIL PROTECTED]>
=



[obm-l] Re: [obm-l] questão IME

2002-11-03 Thread yurigomes

Sejam 2i+1,2i+3,...,2j+1 os termos da PA, com iAlguem pode me ajudar com esta questão do IME do ano de 1997-1998?
>
>Uma soma finita de números inteiros consecutivos, ímpares, positivos ou
negativos,
>é igual a 7^3 (7 elevado ao cubo).
>Determine os termos desta soma.
>
>
>Obrigado.
>

[]'s, Yuri
ICQ: 64992515


--
Use o melhor sistema de busca da Internet
Radar UOL - http://www.radaruol.com.br



=
Instruções para entrar na lista, sair da lista e usar a lista em
http://www.mat.puc-rio.br/~nicolau/olimp/obm-l.html
O administrador desta lista é <[EMAIL PROTECTED]>
=



[obm-l] Re: [obm-l] IME 2003

2002-11-10 Thread yurigomes

   Essa era só perceber que 20 + 14sr(2)=(2+sr(2))^3. Logo, a expressão
é igual a (2+sr(2))+ (2-sr(2))=4.

-- Mensagem original --

>Esta questão é da prova do IME que foi realizada nesta semana que passou.
>Alguém poderia me dar uma ajuda.
>
>
>Qual a melhor forma de resolver exercícios em que se tem que demonstrar
ou
>provar as coisas, tipo essas questões do IME ?
>
>Obrigado pela ajuda.
>Wander
>

[]'s, Yuri
ICQ: 64992515


--
Use o melhor sistema de busca da Internet
Radar UOL - http://www.radaruol.com.br



=
Instruções para entrar na lista, sair da lista e usar a lista em
http://www.mat.puc-rio.br/~nicolau/olimp/obm-l.html
O administrador desta lista é <[EMAIL PROTECTED]>
=



[obm-l] Re: [obm-l] questão 2 - IME 2003

2002-11-10 Thread yurigomes
 Um número é igual ao seu módulo sss ele é maior ou igual a 0. Logo log(12x^3
- 19x^2 +8x) >= 0 
   12x^3 - 19x^2 +8x >= 10^0 = 1
   12x^3 - 19x^2 +8x - 1 >= 0
   (x-1)(x-1/3)(x-1/4) >= 0
 Analisando o sinal dessa função, deve ser 1/4 <= x <= 1/3 ou x >= 1 
   

-- Mensagem original --

>Determine os valores de x que satisfazem a equação:
>
>| log(12x^3 - 19x^2 +8x) | = log(12x^3 -19x^2 +8x),  onde |   | siginica
>módulo.
>
>Obrigado.
>Wander
>

[]'s, Yuri
ICQ: 64992515


--
Use o melhor sistema de busca da Internet
Radar UOL - http://www.radaruol.com.br



=
Instruções para entrar na lista, sair da lista e usar a lista em
http://www.mat.puc-rio.br/~nicolau/olimp/obm-l.html
O administrador desta lista é <[EMAIL PROTECTED]>
=



[obm-l] Re: [obm-l] questão 4 - IME 2003

2002-11-10 Thread yurigomes
  tg(3a) = (tg(a)+tg(2a))/(1-tg(a).tg(2a)), donde
  tg(a) + tg(2a) = tg(3a).(1-tg(a).tg(2a)), e assim queremos
  tg(3a).(1-tg(a).tg(2a)) = 2.tg(3a) sss
  tg(3a)(1-tg(a).tg(2a) - 2) = 0 sss
  tg(3a)( tg(a).tg(2a) + 1) = 0
 Caso i): tg(3a)=0 
  As soluções são a= 0 e a= pi/3
 Caso ii): 1 + tg(a).tg(2a)= 0   
  Mas tg(2a) = (tg(a) + tg(a))/(1- tg(a).tg(a)) = 
  2tg(a)/(1-tg(a)^2) 
 Deve ser então tg(a)^2 = -1, absurdo.
 
-- Mensagem original --

>Resolva a equação:
>
>tg(a) + tg(2a) = 2 . tg(3a)  ,  sabendo-se que a pertence a [0,pi/2).
>
>Obrigado.
>Wander.
>
>

[]'s, Yuri
ICQ: 64992515


--
Use o melhor sistema de busca da Internet
Radar UOL - http://www.radaruol.com.br



=
Instruções para entrar na lista, sair da lista e usar a lista em
http://www.mat.puc-rio.br/~nicolau/olimp/obm-l.html
O administrador desta lista é <[EMAIL PROTECTED]>
=



[obm-l] Re: [obm-l] Terorema de Feuerbach

2002-11-30 Thread yurigomes
 É interessante notar que, dado um triângulo ABC e H seu ortocentro, então
C é ortocentro de ABH. Desse modo, o círculo dos nove pontos de ABH é o
mesmo de ABC, e então esse círculo também é tangente ao incírculo e ex-incírculos
de ABH. O mesmo valendo para ACH e BCH, concluímos que o círculo de Feuerbach
é tangente a 16 outros círculos!!
 A prova pode ser encontrada em Geometry Revisited, do Coxeter.
 

-- Mensagem original --

>Caros amigos, gostaria da demonstração do Terorema  de Feuerbach,
>
>O círculo que atravessa os pés das altitudes de um triângulo toca todos
os
>quatro dos círculos que são tangente aos três lados do triângulo; é interiormente
>tangente ao círculo se inscrito e externamente tangente a cada um dos círculos
>que tocam os lados do triângulo externamente.
>
>
>
>
>Atenciosamente,
>Edmilson
>[EMAIL PROTECTED]
>

[]'s, Yuri
ICQ: 64992515


--
Use o melhor sistema de busca da Internet
Radar UOL - http://www.radaruol.com.br



=
Instruções para entrar na lista, sair da lista e usar a lista em
http://www.mat.puc-rio.br/~nicolau/olimp/obm-l.html
O administrador desta lista é <[EMAIL PROTECTED]>
=



[obm-l] Re: [obm-l] Resultado da OBM

2002-12-06 Thread yurigomes
 Jah saiu.

-- Mensagem original --

>Quando sai o resultado da OBM no site?
>Issao.
>
>___
>Busca Yahoo!
>O melhor lugar para encontrar tudo o que você procura na Internet
>http://br.busca.yahoo.com/
>=
>Instruções para entrar na lista, sair da lista e usar a lista em
>http://www.mat.puc-rio.br/~nicolau/olimp/obm-l.html
>O administrador desta lista é <[EMAIL PROTECTED]>
>=
>

[]'s, Yuri
ICQ: 64992515


--
Use o melhor sistema de busca da Internet
Radar UOL - http://www.radaruol.com.br



=
Instruções para entrar na lista, sair da lista e usar a lista em
http://www.mat.puc-rio.br/~nicolau/olimp/obm-l.html
O administrador desta lista é <[EMAIL PROTECTED]>
=



[obm-l] Re: [obm-l] Olimpíadas ao redor do mundo

2003-03-27 Thread yurigomes
 3x^2 + x= 4y^2 + y sss 3(x^2 - y^2)=y^2-(y-x) sss (x-y)(3(x+y)+1)=y^2 sss
(x-y)(3x+3y+1)=y^2. Pronto, agora sendo d=mdc(x,y), mostre que d=1, donde
os (x-y, 3x+3y+1)=1, e como esse produto eh um QP, deve ser x-y tbm um QP.


-- Mensagem original --

>Apanhei nesses exercicios...quem souber e puder resolvê-los ou dar uma

>sugestão me ajudará muito.
>1)Para os inteiros positivos x e y é verdadeira a igualdade 3x^2+x=4y^2+y.
>
>Mostre que x-y é um quadrado perfeito.
>2) Determine o número primo p para o qual o número 1+p+p^2+p^3+p^4 é um

>quadrado perfeito.
>Saudações a todos os homens de paz e abaixo os invasores.
> Korshinói
>

[]'s, Yuri
ICQ: 64992515


--
Use o melhor sistema de busca da Internet
Radar UOL - http://www.radaruol.com.br



=
Instruções para entrar na lista, sair da lista e usar a lista em
http://www.mat.puc-rio.br/~nicolau/olimp/obm-l.html
O administrador desta lista é <[EMAIL PROTECTED]>
=


[obm-l] Re: [obm-l] Re: [obm-l] f(f(x))_=_x^2_-_1996_é_impossível

2003-06-07 Thread yurigomes

Oi Ricardo. Vc não pode fazer isso, já que não existe garantia de que f
é derivável.
Abraços,
 Yuri
-- Mensagem original --

>diretamente da lista...
>
>f(f(x))=x^2-1996..(1)
>derivando:
>f '(f(x)).f '(x)=2x ..(2)
>x^2-1996=(-x)^2-1996, entao:
>f(f(-x))=f(f(x))=x^2-1996, derivando:
>f '(f(-x)).f '(-x).(-1)=2x --> -f '(f(x)).f '(-x)=2x --> f '(x)=-f '(-x)
>f '(0)=-f '(0) --> f '(0)=0
>fazendo f(x)=0 em (2) temos f '(0).f '(x)=2x=0 --> x=0 --> f(0)=0.
>Mas de (1) --> f(f(0))=-1996 usando f(0)=0 chegamos a f(0)=-1996.
>Logo a tal funcao nao existe.
>
>
>  - Original Message - 
>  From: Bruno Lima 
>  To: [EMAIL PROTECTED] 
>  Sent: Friday, June 06, 2003 7:18 PM
>  Subject: Re: [obm-l] f(f(x))_=_x^2_-_1996_é_impossível
>
>
>  Provar que não existe nenhuma função f: R -> R tal que:
>  f(f(x)) = x^2 - 1996.
>
>
>  Desculpem a besteira...F não tem pontos fixos !
>
>
>
>
>--
>  Yahoo! Mail 
>  Mais espaço, mais segurança e gratuito: caixa postal de 6MB, antivírus,
>proteção contra spam.
>

[]'s, Yuri
ICQ: 64992515


--
Use o melhor sistema de busca da Internet
Radar UOL - http://www.radaruol.com.br



=
Instruções para entrar na lista, sair da lista e usar a lista em
http://www.mat.puc-rio.br/~nicolau/olimp/obm-l.html
=


[obm-l] Re: [obm-l] Re: Off topic-indignação( Que maluquice!!! )

2003-06-04 Thread yurigomes
  Concordo totalmente com a opinião do Frederico. Quem entra na lista tem,
nem que seja mínimo, um interesse em desafios de matemática. E esses muitas
vezes não conseguimos sequer entender o enunciado na primeira vez que lemos.

  Mas isso não é razão pra uma pessoa se revoltar contra a lista e contra
todos que tentam manter ela em um nível elevado (em especial, os professores)
e alegar que não gosta dela pq recebe muitos emails dos quais não se entende
nada! Entrou na chuva é pra se molhar!
  Se no momento a pessoa não tem essa capacidade de aprendizado, retire-se
da lista, estude e volte depois, agora não venha atrapalhar as pessoas que
têm interesse em matemática com emails abslutamente inúteis.
 Espero que a lista volte ao que era antes.
Abraços,
 Yuri
-- Mensagem original --

>Não sei o que se passa com os membros dessa lista, melhor dizendo, com

>alguns deles. Tão absurda a mensagem indignada de um sujeito que se inscreve
>
>numa lista de Problemas das Olimpíadas de Matemática e quer que as mensagens
>
>tratem de temas cotidianos e simplórios, ainda, de forma "didática", quanto
>
>a resposta de um outro que não soube medir as palavras. Causa-me profunda
>
>indignação as duas atitudes e, talvez ainda mais a primeira. Mais uma vez
>
>sou forçado a dizer que as pessoas andam perdendo a noção. Fico entristecido
>
>com a retirada de nosso caro colega, pois sei que será uma inestimável

>perda. Não farei o mesmo pois, apesar desses últimos incidentes, acho a

>lista muito proveitosa e espero que O Nicolau tome as providências 
>necessárias para colocar ordem nessa lista.
>
>Sem mais, encerro dizendo que o respeito à ciência, em especial nesse caso
>à 
>Matemática, é essencial. Vamos pensar um pouco mais antes de escrever 
>mensagens para essa lista.
>
>Frederico.
>
>
>>From: Carlos César de Araújo <[EMAIL PROTECTED]>
>>Reply-To: [EMAIL PROTECTED]
>>To: <[EMAIL PROTECTED]>
>>Subject: [obm-l] Re: [obm-l] Re: [obm-l] Off topic-indignação
>>Date: Tue, 3 Jun 2003 01:49:50 -0300
>>
>>Prezados colegas,
>>
>>Após a leitura de respostas como essas, RETIRO-ME desta lista. Chamem-me
>>quando houver um pouco mais de moderação.
>>
>>
>>- Original Message -
>>From: "+ BRiSSiU +" <[EMAIL PROTECTED]>
>>To: <[EMAIL PROTECTED]>
>>Sent: Tuesday, June 03, 2003 12:58 AM
>>Subject: [obm-l] Re: [obm-l] Off topic-indignação
>>
>>
>>MessagePorque você não vai se foder?
>>
>>Vai filosofar em outro lugar, rapaz.
>>
>>Se você não é capaz de entender a notação matemática aqui utilizada, 
>>aprenda
>>e volte depois.
>>
>>Se não gosta dos enunciados diretos, monte um enredo para si próprio,
mas
>>não nos encha o saco com suas abobrinhas.
>>
>>
>>+ BRiSSiU +
>>   - Original Message -
>>   From: J.Paulo roxer ´til the end
>>   To: [EMAIL PROTECTED]
>>   Sent: Monday, June 02, 2003 8:30 PM
>>   Subject: [obm-l] Off topic-indignação
>>
>>
>>   Eu queria q nesta lista,os assuntos fossem tratados de forma mais
>>objetiva.Não entendo absolutamente nada do que vcs dizem.Estudo,continuo
>>estudando,mas não "entra" essas coisas.
>>   Se fosse só eu,eu poderia fazer uma auto-denominação ao me chamar de
>>estúpido mesmo,mas não é.Tenho mandado algumas questões que vcs põem,
para
>>amigos e eles não entendem.Aliás,é raro alguém na net gostar de tratar
de
>>assuntos como matemática e filosofia.
>>
>>   Em todos meus anos de vida escolar,meus colegas inegavelmente não sabiam
>>do que o professor falava.
>>   É sempre uma explicação mal feita,superficial,que não visa o
>>engrandecimento intelectual do aluno.
>>   Já sei que matemática é até importante,mas deve ser ensinada de forma
>
>>mais
>>concisa,tendo um objetivo.
>>   Não adianta explicar ou fazer uma questão que não se sabe pra que 
>>serve.De
>>nada adianta dar explicações "x" e "y" sem saber a razão que leva a isso.
>>   É como dizer:"Siga por esta rua,vá andando".
>>
>>   Vou tomar como exemplo,a questão colocada por um integrante da
>>lista.Mostre que n! <= (n/2+1)^(n-1), ocorrendo desigualdade estrita para
>>n>=3.
>>
>>   Qual o objetivo?Um software pode ser construído usando isso?Um carro,uma
>>casa,um tubo de ensaio pode ser feito usando essa questão?
>>
>>   É preciso dizer:Nessa questão,usa-se tal fórmula.Nessa outra,usa-se
essa
>>fórmula.Nenhum livro de matemática é coeso,nenhum relata as origens,o

>>porquê
>>e o objetivo (exemplos usados no dia-a-dia)
>>   Nas minhas aulas de física,chega a me dar sono de tanto tédio.
>>
>>   Sinto muito por estarem lendo este e-mail,mas não sei onde colocar

>>minhas
>>dúvidas.Estou em fase de vestibular,vou passar mesmo que seja sem saber
>>mat. e fís. e penso em fazer uma monografia ou algo do tipo que denuncie
>a
>>extrema incapacidade dos professores de matemática e física de
>>ensinar.Nenhum é conciso,prático e possuidor de senso crítico.Apenas
>>aceitam,aprendem e não sabem transmitir o que aprenderam.
>>   Uma coisa é aprender,a outra,é a arte de ser humilde e transmitir da
>
>>forma
>>mais simples possível.
>>
>>   Se alguém

[obm-l] Re: [obm-l] Problema de algum ano do IME

2003-06-07 Thread yurigomes
Oi Leo, 
 Smepre que aparecerem expoentes que são potências de 2 consecutivas, um
argumento que podemos fazer é ver o que acontece qdo multiplicamos a expressão
por um valor que faz ela se reduzir a uma expressão menor. No caso desse
problema, seja
 T=[1+((1+i)/2)].[1+((1+i)/2)^2].[1+((1+i)/2)^4][1+((1+i)/2)^((2)^n)]
Então, usando que (a-b)(a+b)=a^2^-b^2, temos que 
 [1-((1+i)/2))].T= 
 = 
[1-((1+i)/2))].[1+((1+i)/2)].[1+((1+i)/2)^2].[1+((1+i)/2)^4][1+((1+i)/2)^((2)^n)]=
 =[1-((1+i)/2)^2].[1+((1+i)/2)^2].[1+((1+i)/2)^4][1+((1+i)/2)^((2)^n)]=
 = [1-((1+i)/2)^4].[1+((1+i)/2)^4][1+((1+i)/2)^((2)^n)]=
 = ... =
 = [1-((1+i)/2)^(2^n)].[1+((1+i)/2)^(2^n)]=
 = [1+((1+i)/2)^(2^(n+1))]
 Logo, 
  T= [1+((1+i)/2)^(2^(n+1))]/[1-((1+i)/2))]
 Ainda podemos simplificar a fórmula acima. Para n=1, temos
 ((1+i)/2)^(2^(n+1))=((1+i)/2)^(2^2)=[((1+i)/2)^2]^2=
 = (i/2)^2= -1/4. 
 Então T= (1- 1/4)/[1-((1+i)/2))]= 3(1+i)/4.
 Para n >=2, temos   
 ((1+i)/2)^(2^(n+1))=[((1+i)/2)^(2^2)]^(2^(n-1))=
 = (-1/4)^(2^(n-1))= 1/2^(2^n) e daí
 T= (1+ 1/2^(2^n))/((1-i)/2)= (1+ 1/2^(2^n)).(1+i).
 Abraços, 
 Yuri   
-- Mensagem original --

>Será que alguém poderia resolver o seguinte problema:
>Calcule: [1+((1+i)/2)].[1+((1+i)/2)^2].[1+((1+i)/2)^4][1+((1+i)/2)^((2)^n)]
>i = (-1)^(1/2).
>

[]'s, Yuri
ICQ: 64992515


--
Use o melhor sistema de busca da Internet
Radar UOL - http://www.radaruol.com.br



=
Instruções para entrar na lista, sair da lista e usar a lista em
http://www.mat.puc-rio.br/~nicolau/olimp/obm-l.html
=


[obm-l] Re: [obm-l] f(f(x)) = x^2 - 1996 eh impossivel

2003-06-08 Thread yurigomes

Na verdade, o item 2 pode ser garantido, jah que f é sobrejetiva.

-- Mensagem original --

>Caros Gugu, Marcelo e Ricardo:
>
>Muito obrigado pela atencao que voces eram ao problema.
>
>Ricardo: essa solucao que voce viu na lista tem dois problemas:
>1. Supoe que f eh diferenciavel, o que nao faz parte do enunciado.
>2. A uma certa altura, voce escreve: "agora fazemos f(x) = 0" soh que nada
>garante que 0 pertenca a imagem de f.
>
>Sobre o primeiro ponto, considere a seguinte funcao f: R -> R:
>f(x) = 1, se x eh racional e negativo
>f(x) = 2, se x eh irracional e negativo
>f(x) = 3, se x eh >= 0.
>
>f nao eh diferenciavel (nem continua) em nenhum ponto de (-infinito , 0).
>
>No entanto, f(f(x)) = 3 para todo x real.
>
>De qualquer forma, obrigado pelo interesse.
>
>Um abraco,
>Claudio.
>
>=
>Instruções para entrar na lista, sair da lista e usar a lista em
>http://www.mat.puc-rio.br/~nicolau/olimp/obm-l.html
>=
>

[]'s, Yuri
ICQ: 64992515


--
Use o melhor sistema de busca da Internet
Radar UOL - http://www.radaruol.com.br



=
Instruções para entrar na lista, sair da lista e usar a lista em
http://www.mat.puc-rio.br/~nicolau/olimp/obm-l.html
=


[obm-l] Re: [obm-l] Problema de algum ano do IME

2003-06-08 Thread yurigomes

Eh verdade. Eu coloquei o sinal errado. Eh menos mesmo.
 A indução eh pra ver que de dois em dois fatores, a expressão vai diminuindo
de tamanho.
 Ateh mais, 
 Yuri
-- Mensagem original --

>Yuri, como to estudando pro ITA, IME tb me interessa...
>comecei a ver sua resposta, teve uma passagem que me intrigou..
>
> = [1-((1+i)/2)^(2^n)].[1+((1+i)/2)^(2^n)]=
> = [1+((1+i)/2)^(2^(n+1))]
>
>Aqui a ultima linha nao seria [1-((1+i)/2)^(2^(n+1))]??
>não entendi pq vc colocou positivo, é algum erro de atencao meu??
>
>ai o resto é só indução matematica certo?
>
>[]s
>Ariel
>
>*** MENSAGEM ORIGINAL ***
>
>As  21:53 de 7/6/2003 [EMAIL PROTECTED] escreveu:
>
>>Oi Leo, 
>> Smepre que aparecerem expoentes que são potências de 2 consecutivas,
um
>>argumento que podemos fazer é ver o que acontece qdo multiplicamos a
>>expressão
>>por um valor que faz ela se reduzir a uma expressão menor. No caso desse
>>problema, seja
>> T=[1+((1+i)/2)].[1+((1+i)/2)^2].[1+((1+i)/2)^4][1+((1+i)/2)^((2)^n)]
>>Então, usando que (a-b)(a+b)=a^2^-b^2, temos que 
>> [1-((1+i)/2))].T= 
>> =
>>[1-((1+i)/2))].[1+((1+i)/2)].[1+((1+i)/2)^2].[1+((1+i)/2)^4][1+((1+i)/2)^((2)^n)]=
>> =[1-((1+i)/2)^2].[1+((1+i)/2)^2].[1+((1+i)/2)^4][1+((1+i)/2)^((2)^n)]=
>> = [1-((1+i)/2)^4].[1+((1+i)/2)^4][1+((1+i)/2)^((2)^n)]=
>> = ... =
>> = [1-((1+i)/2)^(2^n)].[1+((1+i)/2)^(2^n)]=
>> = [1+((1+i)/2)^(2^(n+1))]
>> Logo, 
>>  T= [1+((1+i)/2)^(2^(n+1))]/[1-((1+i)/2))]
>> Ainda podemos simplificar a fórmula acima. Para n=1, temos
>> ((1+i)/2)^(2^(n+1))=((1+i)/2)^(2^2)=[((1+i)/2)^2]^2=
>> = (i/2)^2= -1/4. 
>> Então T= (1- 1/4)/[1-((1+i)/2))]= 3(1+i)/4.
>> Para n >=2, temos   
>> ((1+i)/2)^(2^(n+1))=[((1+i)/2)^(2^2)]^(2^(n-1))=
>> = (-1/4)^(2^(n-1))= 1/2^(2^n) e daí
>> T= (1+ 1/2^(2^n))/((1-i)/2)= (1+ 1/2^(2^n)).(1+i).
>> Abraços, 
>> Yuri   
>>-- Mensagem original --
>>
>>>Será que alguém poderia resolver o seguinte problema:
>>>Calcule:
>>[1+((1+i)/2)].[1+((1+i)/2)^2].[1+((1+i)/2)^4][1+((1+i)/2)^((2)^n)]
>>>i = (-1)^(1/2).
>>>
>>
>>[]'s, Yuri
>>ICQ: 64992515
>>
>>
>>--
>>Use o melhor sistema de busca da Internet
>>Radar UOL - http://www.radaruol.com.br
>>
>>
>>
>>=
>>Instruções para entrar na lista, sair da lista e usar a lista em
>>http://www.mat.puc-rio.br/~nicolau/olimp/obm-l.html
>>=
>
>
>
>=
>Instruções para entrar na lista, sair da lista e usar a lista em
>http://www.mat.puc-rio.br/~nicolau/olimp/obm-l.html
>=
>

[]'s, Yuri
ICQ: 64992515


--
Use o melhor sistema de busca da Internet
Radar UOL - http://www.radaruol.com.br



=
Instruções para entrar na lista, sair da lista e usar a lista em
http://www.mat.puc-rio.br/~nicolau/olimp/obm-l.html
=


[obm-l] Re: [obm-l] Re: [obm-l] Problema de algum ano do IME(corrigindo)

2003-06-08 Thread yurigomes
 O Ariel percebeu um erro de sinal. Segue a correção abaixo. 

-- Mensagem original --

>Oi Leo, 
> Sempre que aparecerem expoentes que são potências de 2 consecutivas, um
>argumento que podemos fazer é ver o que acontece qdo multiplicamos a expressão
>por um valor que faz ela se reduzir a uma expressão menor. No caso desse
>problema, seja
> T=[1+((1+i)/2)].[1+((1+i)/2)^2].[1+((1+i)/2)^4][1+((1+i)/2)^((2)^n)]
>Então, usando que (a-b)(a+b)=a^2^-b^2, temos que 
> [1-((1+i)/2))].T= 
> = 
> [1-((1+i)/2))].[1+((1+i)/2)].[1+((1+i)/2)^2].[1+((1+i)/2)^4][1+((1+i)/2)^((2)^n)]=
> =[1-((1+i)/2)^2].[1+((1+i)/2)^2].[1+((1+i)/2)^4][1+((1+i)/2)^((2)^n)]=
> = [1-((1+i)/2)^4].[1+((1+i)/2)^4][1+((1+i)/2)^((2)^n)]=
> = ... =
> = [1-((1+i)/2)^(2^n)].[1+((1+i)/2)^(2^n)]=
> = [1-((1+i)/2)^(2^(n+1))]
> Logo, 
>  T= [1-((1+i)/2)^(2^(n+1))]/[1-((1+i)/2))]
> Ainda podemos simplificar a fórmula acima. Para n=1, temos
> ((1+i)/2)^(2^(n+1))=((1+i)/2)^(2^2)=[((1+i)/2)^2]^2=
> = (i/2)^2= -1/4. 
> Então T= (1-(- 1/4))/[1-((1+i)/2))]= 5(1+i)/4.
> Para n >=2, temos   
> ((1+i)/2)^(2^(n+1))=[((1+i)/2)^(2^2)]^(2^(n-1))=
> = (-1/4)^(2^(n-1))= 1/2^(2^n) e daí
> T= (1- 1/2^(2^n))/((1-i)/2)= (1- 1/2^(2^n)).(1+i).
Veja que essa ultima formula fornece 5(1+i)/4 para n=1. Temos então a solução
geral
  T= (1- 1/2^(2^n)).(1+i).

> Abraços, 
> Yuri   
>-- Mensagem original --
>
>>Será que alguém poderia resolver o seguinte problema:
>>Calcule: [1+((1+i)/2)].[1+((1+i)/2)^2].[1+((1+i)/2)^4][1+((1+i)/2)^((2)^n)]
>>i = (-1)^(1/2).
>>
>
>[]'s, Yuri
>ICQ: 64992515
>
>
>--
>Use o melhor sistema de busca da Internet
>Radar UOL - http://www.radaruol.com.br
>
>
>
>=
>Instruções para entrar na lista, sair da lista e usar a lista em
>http://www.mat.puc-rio.br/~nicolau/olimp/obm-l.html
>=
>

[]'s, Yuri
ICQ: 64992515


--
Use o melhor sistema de busca da Internet
Radar UOL - http://www.radaruol.com.br



=
Instruções para entrar na lista, sair da lista e usar a lista em
http://www.mat.puc-rio.br/~nicolau/olimp/obm-l.html
=


[obm-l] Re: [obm-l] Re: [obm-l] Re: [obm-l] Problema de algum ano do IME(corrigindo)

2003-06-08 Thread yurigomes


-- Mensagem original --

> O Ariel percebeu um erro de sinal. Segue a correção abaixo. 
>
>-- Mensagem original --
>
>>Oi Leo, 
>> Sempre que aparecerem expoentes que são potências de 2 consecutivas,
um
>>argumento que podemos fazer é ver o que acontece qdo multiplicamos a expressão
>>por um valor que faz ela se reduzir a uma expressão menor. No caso desse
>>problema, seja
>> T=[1+((1+i)/2)].[1+((1+i)/2)^2].[1+((1+i)/2)^4][1+((1+i)/2)^((2)^n)]
>>Então, usando que (a-b)(a+b)=a^2^-b^2, temos que 
>> [1-((1+i)/2))].T= 
>> = 
>> [1-((1+i)/2))].[1+((1+i)/2)].[1+((1+i)/2)^2].[1+((1+i)/2)^4][1+((1+i)/2)^((2)^n)]=
>> =[1-((1+i)/2)^2].[1+((1+i)/2)^2].[1+((1+i)/2)^4][1+((1+i)/2)^((2)^n)]=
>> = [1-((1+i)/2)^4].[1+((1+i)/2)^4][1+((1+i)/2)^((2)^n)]=
>> = ... =
>> = [1-((1+i)/2)^(2^n)].[1+((1+i)/2)^(2^n)]=
>> = [1-((1+i)/2)^(2^(n+1))]
>> Logo, 
>>  T= [1-((1+i)/2)^(2^(n+1))]/[1-((1+i)/2))]
>> Ainda podemos simplificar a fórmula acima. Para n=1, temos
>> ((1+i)/2)^(2^(n+1))=((1+i)/2)^(2^2)=[((1+i)/2)^2]^2=
>> = (i/2)^2= -1/4. 
>> Então T= (1-(- 1/4))/[1-((1+i)/2))]= 5(1+i)/4.
>> Para n >=2, temos   
>> ((1+i)/2)^(2^(n+1))=[((1+i)/2)^(2^2)]^(2^(n-1))=
>> = (-1/4)^(2^(n-1))= 1/2^(2^n) e daí
>> T= (1- 1/2^(2^n))/((1-i)/2)= (1- 1/2^(2^n)).(1+i).
>>> Abraços, 
>> Yuri   
>>-- Mensagem original --
>>
>>>Será que alguém poderia resolver o seguinte problema:
>>>Calcule: [1+((1+i)/2)].[1+((1+i)/2)^2].[1+((1+i)/2)^4][1+((1+i)/2)^((2)^n)]
>>>i = (-1)^(1/2).
>>>
>>
>>[]'s, Yuri
>>ICQ: 64992515
>>
>>
>>--
>>Use o melhor sistema de busca da Internet
>>Radar UOL - http://www.radaruol.com.br
>>
>>
>>
>>=
>>Instruções para entrar na lista, sair da lista e usar a lista em
>>http://www.mat.puc-rio.br/~nicolau/olimp/obm-l.html
>>=
>>
>
>[]'s, Yuri
>ICQ: 64992515
>
>
>--
>Use o melhor sistema de busca da Internet
>Radar UOL - http://www.radaruol.com.br
>
>
>
>=
>Instruções para entrar na lista, sair da lista e usar a lista em
>http://www.mat.puc-rio.br/~nicolau/olimp/obm-l.html
>=
>

[]'s, Yuri
ICQ: 64992515


--
Use o melhor sistema de busca da Internet
Radar UOL - http://www.radaruol.com.br



=
Instruções para entrar na lista, sair da lista e usar a lista em
http://www.mat.puc-rio.br/~nicolau/olimp/obm-l.html
=


[obm-l] Re: [obm-l] [E.M.] conjugado de complexos

2003-06-08 Thread yurigomes

Basta vc observar que ~(v.w)=(~v).(~w). Para ver isso, chame  v de a+bi
e w de c+di e faça as contas:
 ~(v.w)= ~(ab-bd+ (ad+bc)i)= ab-bd-(ad+bc)i
  (~v).(~w)=(a-bi)(c-di)= ab-bd-(ad+bc)i
 Logo, aplicando isso n vezes, vc chega ao resultado.
 Ateh mais, 
  Yuri
-- Mensagem original --

>bom, nao sei nenhuma notacao para conjugado... 
>
>consideremos ~z conjugado de z
>
>como provar que ~(z^n)=(~z)^n
>
>é uma questao bobinha, mas nao sei como demonstrar isso no papel
>
>[]s
>Ariel
>
>=
>Instruções para entrar na lista, sair da lista e usar a lista em
>http://www.mat.puc-rio.br/~nicolau/olimp/obm-l.html
>=
>

[]'s, Yuri
ICQ: 64992515


--
Use o melhor sistema de busca da Internet
Radar UOL - http://www.radaruol.com.br



=
Instruções para entrar na lista, sair da lista e usar a lista em
http://www.mat.puc-rio.br/~nicolau/olimp/obm-l.html
=


[obm-l] Re: [obm-l] QUESTÕES INTERESSANTES

2003-07-09 Thread yurigomes
Oi Frederico,
 Gostei das questões! =P

(1)( => ) Suponha A e A^(-1) com entradas inteiras. Então  detA e detA^(-1)
são inteiros. Mas como detA.detA^(-1)= 1, devemos ter detA= +-1.
 ( <= ) Ora, se A= (a  b), então A^(-1)= 1/detA.(d  -b), e assim 
   (c  d)   (-c  a)
como detA= +-1, segue que A^(-1) tem entradas inteiras.

(2) Se vc tem um complexo z= a+ bi, então a rotação de z sobre a origem
de um ângulo A é obtido multiplicando z por cisA= cosA + i.senA. Logo os
outros dois vértices do triângulo são:
(sqrt(3)+ i)cis(120º) e (sqrt(3)+ i)cis(240º).

(3) Temos raiz real dupla se e só se delta= 0 <=>
  b^2= 4ac. Daí sendo b primo, temos b= 2, e assim 2^2= 4ac =>
1= ac. Logo a, c não podem ser primos.

(4) Basta analisar módulo 8. Os quadrados só deixam resto 0, 1 e 4 (módulo
8), Mas 3^x= 1 (mod 8) se x é par e 3^x= 3 (mod 8) se x é ímpar. Logo 3^a+
3^b+ 1 só pode deixar resto
 1+ 1+ 1= 3 (mod 8)
 1+ 3+ 1= 5 (mod 8)
 3+ 3+ 1= 7 (mod 8)
 Assim, 3^a+ 3^b+ 1 nunca pode ser um quadrado perfeito.

(5) Se 1/p= a_0,a_1...a_i b_1...b_T(p) b_1...b_T(p)..., então
   10^i.(1/p)=a_0 a_1...a_i,b_1..b_T(p) b_1..b_T(p)..
10^(i+T(p))(1/p)=a_0 a_1..a_ib_1..b_T(p),b_1..b_T(p)...
 Daí,
(10^(i+T(p))-10^i).(1/p)=k, onde k é inteiro. Logo:
   (10^i)(10^(T(p))- 1)= p.k
  Como p é maior do que 5, temos (p, 10^i)= 1, donde p|(10^T(p)- 1) => 10^(T(p))=
1 (mod p) (*)
 (a) Se T(p)=2, então p|10^2- 1= 99 => p= 3 ou p= 11. Como p>5, temos que
p=11.
 (b) Continuando de (*), temos ord10(mod p)|T(p). Mas é fácil fazer o mesmo
que fizemos acima voltando para concluir que ord10(mod p)= T(p). Daí, T(p)|(p-1),
pois 10^(p- 1)= 1 (mod p), Além disso, R(n)= 111= 1+ 10+ ...+ 10^(n-
1)= (10^n- 1)/9. Logo p|R(n) <=> p|(10^n- 1)/9 <=> p|(10^n- 1) <=> 10^n=
1 (mod p), e o valor mínimo de n satisfazendo essa ultima congruencia é
ord10(mod p)= T(p).

(6) Se a parábola tivesse raizes racionais, deveriamos ter delta= x^2, com
x inteiro. Daí:
b^2- 4.ac= x^2 => x ímpar
 Então b^2= x^2= 1 (mod 8), de modo que 8|(b^2- x^2) => 8|4ac, absurdo,
já que a, c são impares.

   Ateh mais,
  Yuri
   Mensagem original --

>Olá caros colegas da lista, seguem algumas questões que considero muito

>interessantes, para quem quiser se distrair um pouco nessas férias:
>
>(1)   Seja   A   uma matriz 2x2  com entradas inteiras. Mostre que  A tem
>
>inversa " com entradas ingteiras"   se, e somente se,   det(A) = + - 1
.
>(
>Uma das implicações é trivial. )
>
>(2)   Um triângulo equilátero, inscrito numa circunferência de centro na
>
>origem do sistema cartesiano, tem  o   número complexoz= sqrt(3)  +
i
>
>como um de seus vértices. Determine os outros.
>
>(3)  Uma equação quadrática com coeficientes primos pode ter raiz real

>dupla?
>
>(4) Sejam   a   , bnúmeros naturais.  Mostre  que (3^a  + 3^b + 1 )

>
>nunca é um quadrado perfeito.
>
>(5) Seja  p  > 5   um número primo. Neste caso   1/p , o recíproco de
p
>,
>tem por  representação decimal uma dízima periódica. Indiquemos por  T(p)
> o
>número de algarismos que constituem o período.
>(a)  Mostre que existe um único primo   p  tal que  T(p)  = 2 .
>(b) Mostre que  T(p)   é o menor número natural  n   tal   que:  n
|
>
>(p-1) e  p | R_n  ,  em que  R_n = ...1 ( n  dígitos 1 ) .
>Obs:  a | b  quandob   é múltiplo de  a , isto é, existe  q  inteiro
>tal
>que   b = a . q   .
>
>(6) Mostre que se   a , b, c  são inteiros  ímpares   então a eq. 
>ax^2+bx+c=0   não tem raiz racional.
>( Proposto por Eduardo Wagner no 1o encontro da RPM. )
>
>Até a próxima.
>Frederico.
>
>
>>From: "Frederico Reis Marques de Brito" <[EMAIL PROTECTED]>
>>Reply-To: [EMAIL PROTECTED]
>>To: [EMAIL PROTECTED]
>>Subject: Re: [obm-l] Duvida sobre polinomios
>>Date: Wed, 09 Jul 2003 11:08:23 -0300
>>
>>
>>Sim, desde que o polinômio divisor não seja nulo. Existe um resultado,

>>análogo ao Lema da Divisão de Euclides para nos inteiros, que garante
que
>
>>dados   polinõmios   f(x), g(x) , g(x) <> 0,  com coeficientes num corpo
>K
>>( em particular se  K= R = conjunto dos numeros reais )  então existem
e
>
>>são únicos  q(x) e  r(x)  com coeficientes em  K  de tal forma que 
 f(x)
>
>>= g(x) X q(x) + r(x) e " r(x) =0   ougrau(r) < gr(g)" .  A

>>demonstração desse fato é, normalmente, obtida através de indução
>>matemática. Você pode obtê-la, por exemplo, em
>>
>>[1]GONÇALVES, Adilson -  "Introdução Á Álgebra"  -  Projeto Euclides
>-
>>SBM/IMPA.
>>[2]   DOMINGUES, Hygino H. , IEZZI, Gelson - " Álgebra Moderna"-  Atual
>
>>Editora.
>>
>>Observe ainda que a hipótese de que os polinômios tenham coeficientes
num
>
>>corpo ( anel comutativo com elemento neutro do produto e no qual todo

>>elemento não -nulo tenha inverso, ufa!!! ) é absolutamente essencial.
SE
>
>>dividirmos  F(x)= x+1   por   G(x)  = 2   , olhando-os como polinômios
a
>
>>coef. inteiros, não obteremos um quociente com esta propriedade.
>>Frederico.
>>
>>>From: "leonardo mattos" <[EMAIL PROTEC

[obm-l] Re: [obm-l] Combinatoria

2003-07-12 Thread yurigomes

 Oi Marcio, 
  Se eu não me engano, esse problema tem no Problem Solving:
 Seja x_i= número de partidas jogadas até o dia i, inclusive. 
  Como o enxadrista joga no minimo 1 partida por dia e no máximo 11x12=
132 no total, temos 
  1<= a_1< a_2<...< a_77<= 132. Some 20 na desigualdade:
 21<= a_1 + 20<...< a_77 + 20 <= 152. 
 Então, os números a_1, a_2,..., a_77,  a_1 + 20,...,a_77 + 20 estão entre
1 e 152. Como temos 154 números, pelo princípio da casa dos pombos existem
dois deles iguais. Assim, existem dois indices i e j, i!=j, tais que 
  a_i= a_j + 20.
  Ora, isso é equivalente ao enxadrista ter jogado exatamente vinte partidas
entre os dias i+1 e j.  

 Ateh mais, 
 Yuri

-- Mensagem original --

>Nao estou conseguindo fazer a seguinte questao, do livro de combinatoria
>do Morgado:
>Um enxadrista joga partidas de xadrez durante onze semanas consecutivas.
>Sabe-se que ele sempre joga ao menos uma partida por dia, e jamais joga
mais
>de 12 partidas em uma semana. Mostre que existe um periodo de dias consecutivos
>no qual ele joga exatamente 20 partidas.
>Alguem tem alguma dica?
>
>Abracos,
>Marcio
>

[]'s, Yuri
ICQ: 64992515


--
Use o melhor sistema de busca da Internet
Radar UOL - http://www.radaruol.com.br



=
Instruções para entrar na lista, sair da lista e usar a lista em
http://www.mat.puc-rio.br/~nicolau/olimp/obm-l.html
=


[obm-l] Re: [obm-l] Prova da IMO

2003-07-13 Thread yurigomes
 Na verdade ela jah deveria estar, pq o horario da prova aqui no Brasil
foi ontem (sábado) aa noite. Hj aa noite serah realizado o segundo dia.
Ateh agora ainda não achei. Se alguém conseguir, favor mandar para a lista.

 Ateh mais, 
 Yuri

-- Mensagem original --

>Hoje será o primeiro dia de prova da IMO. Será que a prova estará online
>ao final do dia??
>Cícero
>
>
>
>--
>Use o melhor sistema de busca da Internet
>Radar UOL - http://www.radaruol.com.br
>
>
>
>=
>Instruções para entrar na lista, sair da lista e usar a lista em
>http://www.mat.puc-rio.br/~nicolau/olimp/obm-l.html
>=
>

[]'s, Yuri
ICQ: 64992515


--
Use o melhor sistema de busca da Internet
Radar UOL - http://www.radaruol.com.br



=
Instruções para entrar na lista, sair da lista e usar a lista em
http://www.mat.puc-rio.br/~nicolau/olimp/obm-l.html
=


[obm-l] Re: [obm-l] Re: [obm-l] Combinatoria

2003-07-14 Thread yurigomes
 O principio da casa dos pombos (PCP), ou Principio de Dirichlet, na sua
forma mais simples, diz que se vc tem n+1 bolas e quer distribuí-las em
n gavetas, então algumas das gavetas deverá conter no minimo duas bolas.

 Isso eh bem intuitivo. Para provar isso, suponha por absurdo que não. Então
cada gaveta terá no máximo 1 bola. Como temos n gavetas, isso nos dá um
numero maximo de 1+1+...+1= n bolas. Mas nós temos n+1 !! Absurdo. E o resultado
segue. 
  Outra forma do PCP é a seguinte: se agora vc tem nk+ 1 bolas e quer distribui-las
em n gavetas, alguma das gavetas terá no minimo k+ 1 bolas. 
 Veja que a versão anterior é um caso particular desta ultima, com k= 1.
Tente prová-la, é a mesma idéia ( absurdo! ). 
 Ateh mais, 
 Yuri

-- Mensagem original --

>
>Alguem me explica como eh esse principio da casa dos pombos?
>
>obrigado
>
>>From: [EMAIL PROTECTED]
>>Reply-To: [EMAIL PROTECTED]
>>To: [EMAIL PROTECTED]
>>Subject: [obm-l] Re: [obm-l] Combinatoria
>>Date: Sat, 12 Jul 2003 12:22:50 -0300
>>
>>
>>  Oi Marcio,
>>   Se eu não me engano, esse problema tem no Problem Solving:
>>  Seja x_i= número de partidas jogadas até o dia i, inclusive.
>>   Como o enxadrista joga no minimo 1 partida por dia e no máximo 11x12=
>>132 no total, temos
>>   1<= a_1< a_2<...< a_77<= 132. Some 20 na desigualdade:
>>  21<= a_1 + 20<...< a_77 + 20 <= 152.
>>  Então, os números a_1, a_2,..., a_77,  a_1 + 20,...,a_77 + 20 estão
entre
>>1 e 152. Como temos 154 números, pelo princípio da casa dos pombos existem
>>dois deles iguais. Assim, existem dois indices i e j, i!=j, tais que
>>   a_i= a_j + 20.
>>   Ora, isso é equivalente ao enxadrista ter jogado exatamente vinte 
>>partidas
>>entre os dias i+1 e j.
>>
>>  Ateh mais,
>>  Yuri
>>
>>-- Mensagem original --
>>
>> >Nao estou conseguindo fazer a seguinte questao, do livro de 
>>combinatoria
>> >do Morgado:
>> >Um enxadrista joga partidas de xadrez durante onze semanas consecutivas.
>> >Sabe-se que ele sempre joga ao menos uma partida por dia, e jamais joga
>>mais
>> >de 12 partidas em uma semana. Mostre que existe um periodo de dias 
>>consecutivos
>> >no qual ele joga exatamente 20 partidas.
>> >Alguem tem alguma dica?
>> >
>> >Abracos,
>> >Marcio
>> >
>>
>>[]'s, Yuri
>>ICQ: 64992515
>>
>>
>>--
>>Use o melhor sistema de busca da Internet
>>Radar UOL - http://www.radaruol.com.br
>>
>>
>>
>>=
>>Instruções para entrar na lista, sair da lista e usar a lista em
>>http://www.mat.puc-rio.br/~nicolau/olimp/obm-l.html
>>=
>
>_
>MSN Hotmail, o maior webmail do Brasil.  http://www.hotmail.com
>
>=
>Instruções para entrar na lista, sair da lista e usar a lista em
>http://www.mat.puc-rio.br/~nicolau/olimp/obm-l.html
>=
>

[]'s, Yuri
ICQ: 64992515


--
Use o melhor sistema de busca da Internet
Radar UOL - http://www.radaruol.com.br



=
Instruções para entrar na lista, sair da lista e usar a lista em
http://www.mat.puc-rio.br/~nicolau/olimp/obm-l.html
=


[obm-l] Re: [obm-l] Resultado do Brasil na IMO

2003-07-16 Thread yurigomes

  Parabéns a todos da equipe, 
  Em comparação com a última IMO, os resultados foram parecido. De fato,
novamente conseguimos uma e "cheiramos" mais 3 medalhas de prata!
 Novamente parabéns a todos: Davi, Larissa, Alex, Hirama, Samuel (ele fez
a sexta!) e Fabinho. 
 Mas Nicolau, quem foi o argentino medalha de ouro? Lucas Rearte?

 Ateh mais,
 Yuri
-- Mensagem original --

>Os totais minimos para bronze, prata e ouro foram respectivamente
>13, 19 e 29. Assim o resultado do nosso time foi o seguinte:
>
>  1 2 3 4 5 6 Total
>
>BRA1 = Alex   7 3 0 7 1 0  18  (BRONZE)
>BRA2 = Samuel 2 2 0 7 0 7  18  (BRONZE)
>BRA3 = Rafael 0 1 0 7 3 0  11  (MH)
>BRA4 = Larissa1 0 0 7 0 1   9  (MH)
>BRA5 = Fabio  7 3 0 7 1 1  19  (PRATA)
>BRA6 = David  2 3 0 7 4 1  17  (BRONZE)
>
>   Total 1912 042 910  92
>
>Tr^es estudantes fizeram 42 pontos, um da China e dois do Vietnam.
>O pais com o maior total de pontos foi a Bulgaria, significativamente
>aa frante da China, 2a colocada. A Argentina conquistou uma medalha de
ouro
>(com 29 pontos) mas ficou 1 ponto atras do Brasil no total de pontos.
>
>As proximas IMOs ser~ao na Grecia (2004), Mexico (2005), Eslovenia (2006)
>e Vietnam (2007).
>
>[]s, Nicolau
>=
>Instruções para entrar na lista, sair da lista e usar a lista em
>http://www.mat.puc-rio.br/~nicolau/olimp/obm-l.html
>=
>

[]'s, Yuri
ICQ: 64992515


--
Use o melhor sistema de busca da Internet
Radar UOL - http://www.radaruol.com.br



=
Instruções para entrar na lista, sair da lista e usar a lista em
http://www.mat.puc-rio.br/~nicolau/olimp/obm-l.html
=


[obm-l] Re: [obm-l] IMO - P1

2003-07-19 Thread yurigomes

Oi Marcio,
 Soh hj eu li seu email, depois que eu tbm consegui fazer a questão. 
 Tem apenas um detalhe que vc não observou: os t_i´s devem ser distintos,
pq senão os dois conjuntos seriam iguais. 
 Seguindo a sua notação, sendo D_i=(D+ t_i)U(t_i- D), temos |D_i|<= 2.5050.
O t_(i+1) deve ser escolhido em 
   T = S\(S_1 U...U S_i U {t_1, t_2,...,t_i})
  Olha como o problema é impressionante: para garantir que t_100 pode ser
escolhido, devemos ter T não-vazio. Ora, 
|S_1 U...U S_99 U {t_1, t_2,...,t_99}|<= |S_1|+...+ |S_99|+ 99<=
  99.2.5050+ 99= 00+ 99= 99 < 100  ()
   Os números foram muitos bem escolhidos, e o problema ainda não perdeu
a elegância com números feios! NOvamente, parabéns Gugu.
  Ateh mais, 
  Yuri
-- Mensagem original --

>Acho que consegui fazer  o 1o. Confiram ai e vejam se tem algum furo.
>O
>2o eu realmente nao estou conseguindo.. Estou com alguma esperanca de fazer
>o 5.. (o 3 eu tentei tmb, mas minhas contas estao muito grandes). Mandem
>seus comentarios sobre a prova!
>P1:
>Note que (Ai inter Aj) != vazio sse existirem m,n tais que a_m + t_i
>=
>a_n + t_j , i.e, a_m - a_n = t_j - t_i.
>Vamos construir os t's indutivamente garantindo que isso nao acontece.
>Existem binomial (101,2) = 5050 diferencas possiveis no conjunto A. Chame
>de
>D={D1,D2,...D5050} o conjunto dessas diferencas (claro que algumas delas
>podem ser iguais, mas temos |D| <= 5050).
>1. Escolha um t1 qualquer de S.
>2. Agora quero garantir que t2-t1 e t1-t2 nao estao em D. Para isso, basta
>escolher um elemento de S que nao esteja em
>X1 = {t1+D1,t1+D2,...,t1+D5050}U{t1-D1, t1-D2,...,t1-D5050}. (pq se t2-t1
>esta em D, entao t2=t1+Dk para algum k).
>Isso eh facil pq |X1|<=2.5050 < |S|.
>
>3. Agora vou escolher t3 em S garantindo que t3-t1, t1-t3, t3-t2, t2-t3
nao
>estao em D.
>Para isso, t3 nao pode estar em X1 e tmb nao pode estar em
>X2 = {t2+D1,t2+D2,...,t2+D5050}U{t2-D1, t2-D2,...,t2-D5050}.
>Isso eh facil, pq |X1 U X2| <= 4.5050 < |S|
>
>Em geral, depois de escolhidos t1,t2,...,t_k-1, vou escolher t_k em S de
>modo que ele nao esteja em nenhum dos conjuntos X1,X2,...,X_(k-1).
>Para k<=100, isso eh sempre possivel, pq |X1 U X2 U ... U X_(k-1)| <=
>2*(k-1)*5050 <= 2*99*5050 = 00 < 10^6 = |S|.
>(obs: X_s = {ts + D}U{ts-D}, na notacao usuao de x+A onde x eh um elemento
>e
>A um conjunto).
>
>
>Pronto. Foram escolhidos 100 t's tal que nao existe uma quadrupla (m,n,i,j)
>tq a_m - a_n = t_j - t_i. (pois t_j - t_i esta sempre fora de D), e portanto
>nunca se tem a_m + t_i = a_n + t_j, ou seja, as intersecoes sao todas vazias
>de fato.
>
>Abracos.
>
>
>
>- Original Message -
>From: <[EMAIL PROTECTED]>
>To: <[EMAIL PROTECTED]>
>Cc: <[EMAIL PROTECTED]>; <[EMAIL PROTECTED]>
>Sent: Monday, July 14, 2003 3:38 PM
>Subject: [obm-l] Problemas da IMO
>
>
>>
>>
>> Prova da IMO retirada do Site http://www.mathlinks.go.ro/
>>
>> O Problema 1 é nois que mandou...
>>
>>
>> First Day - 44th IMO 2003 Japan
>>
>> 1. Let A be a 101-element subset of the set S={1,2,3,...,100}. Prove
>that
>> there exist numbers t_1, t_2, ..., t_{100} in S such that the sets
>>
>> Aj = { x + tj | x is in A } for each j = 1, 2, ..., 100
>>
>> are pairwise disjoint.
>>
>>
>> 2. Find all pairs of positive integers (a,b) such that the number
>>
>> a^2 / ( 2ab^2-b^3+1) is also a positive integer.
>>
>> 3. Given is a convex hexagon with the property that the segment connecting
>the
>> middle points of each pair of opposite sides in the hexagon is  sqrt(3)
>/
>2
>> times the sum of those sides' sum.
>>
>> Prove that the hexagon has all its angles equal to 120.
>>
>>
>> Second Day - 44th IMO 2003 Japan
>>
>> 4. Given is a cyclic quadrilateral ABCD and let P, Q, R be feet of the
>> altitudes from D to AB, BC and CA respectively. Prove that if PR = RQ
then
>the
>> interior angle bisectors of the angles < ABC and < ADC are concurrent
on
>AC.
>>
>> 5. Let x1 <= x2 <= ... <= xn be real numbers, n>2.
>>
>> a) Prove the following inequality:
>>
>> (sum  ni,j=1 | xi - xj | ) 2 <= 2/3 ( n^2 - 1 )sum ni,j=1 ( xi - xj)^2
>>
>> b) Prove that the equality in the inequality above is obtained if and
only
>if
>> the sequence (xk) is an arithemetical progression.
>>
>> 6. Prove that for each given prime p there exists a prime q such that
>n^p - p
>> is not divisible by q for each positive integer n.
>>
>>
>>
>> -
>> This mail sent through IMP: http://horde.org/imp/
>> =
>> Instruções para entrar na lista, sair da lista e usar a lista em
>> http://www.mat.puc-rio.br/~nicolau/olimp/obm-l.html
>> =
>>
>
>=
>Instruções para entrar na lista, sair da lista e usar a lista em
>http://www.mat.puc-rio.br/~nicolau/olimp/obm-l.html
>

[obm-l] Re: [obm-l] sequências....

2003-07-21 Thread yurigomes
  Oi Crom,
 Aih vão as soluções:

 1) Vamos mostrar por indução. Para n=1, temos a_1^3=a_1^2 => a_1=0 ou a_1=1.OK.
Além disso, 1+ 8.a_1 é quadrado perfeito.
 Suponha por indução que a_1, ...a_(n-1) sejam inteiros e que 1+ 8(a_1+...+a_(n-1)).(
Vc vai jah perceber pq essa ultima condição). Logo
  a_1^3+...+a_n^3= (a_1+...+a_n)^2 =>
(a_1^3+...+a_(n-1)^3)+ a_n^3= (a_1+...+a_(n-1))^2+ 2.a_n.(a_1+...+a_(n-1))+
a_n^2 =>
=>a_n^3= 2.a_n.(a_1+...+a_(n-1))+ a_n^2 =>
=> a_n=0 ou  a_n^2= 2.(a_1+...+a_(n-1))+ a_n =>
=>  a_n^2- a_n- 2.(a_1+...+a_(n-1))=0.
   delta= 1+ 8(a_1+...+a_(n-1)), que por indução, é quadrado perfeito (e
ímpar, como se pode ver).
 Assim, a_n= [1+- sqr(1+ 8(a_1+...+a_(n-1))]/2, que em qq dos casos é inteiro!
Para completar a indução, mostremos que 1+8(a_1+...+a_n) tbm é quadrado
perfeito.
 De fato:
 a_n^2= 2.(a_1+...+a_(n-1))+ a_n =>  1+8(a_1+...+a_n)= 4.a_n^2+ 4.a_n+ 1=
(2.a_n+ 1)^2, e assim a indução está completa.

 2) Escolha a_1= 1, a_2= -1, a_3= 2, a_4= -2, a_5= 3 e a_6= -3. Logo: muitos
termos se cancelam, e o problema se reduz a achar quatro reais a, b, c,
d t.q.
 (x- a)(x- b)(x- c)(x- d)= (x+ a)(x+ b)(x+ c)(x+ d).
  Vc pode observar que essa igualdade se reduz a um polinômio de grau 3,
da forma
   p(x)= r.x^3+ s.x
x=0 satisfaz tal equação, e p(x)/x= r.x^2+ s, que tem duas raízes reais
sss r.s <0. Veja que r= (a+b+c+d) e s= abcd(1/a+ 1/b+ 1/c+ 1/d). Tomando
a, b, c, d tais que esse produto dê negativo( por exemplo: a= 4, b=5, c=6
e d=-7, temos r>0 e s<0), as raízes 0, x_1 e x_2 de p(x) satisfazem a equação
inicial. Além disso, -x_1 e -x_2 tbm satisfazem, como é fácil de se verificar.
Daí, temos exatamente cinco soluções distintas.



-- Mensagem original --


>1)A sequência de números reais ( a_1,a_2,,a_2000) satisfaz a condição:
>a_1^3+a_2^3++a_n^3=(a_1+a_2++a_n)^2 para todo n, 1<=n<=2000. Mostre
>
>que todo elemento da sequência é um número inteiro.
>2) Prove a existência de números reais distintos a_1,a_2,...a_10 tais que
>a
>equação
>(x-a_1)(x-a_2)(x-a_10)=(x+a_1)(x+a_2).(x+a_10), possui exatamente
>5
>raízes distintas...
>Qualquer ajuda, eu agradeço.
>   Crom
>

[]'s, Yuri
ICQ: 64992515


--
Use o melhor sistema de busca da Internet
Radar UOL - http://www.radaruol.com.br



=
Instruções para entrar na lista, sair da lista e usar a lista em
http://www.mat.puc-rio.br/~nicolau/olimp/obm-l.html
=


[obm-l] Re: [obm-l] Resultado do Brasil na IMO

2003-07-23 Thread yurigomes
Oi João,
Naum achei. Eh exatamente nesse link??

-- Mensagem original --

>At 21:57 16/7/2003 -0300, you wrote:
>>Os totais minimos para bronze, prata e ouro foram respectivamente
>>13, 19 e 29. Assim o resultado do nosso time foi o seguinte:
>>
>>   1 2 3 4 5 6 Total
>>
>>BRA1 = Alex   7 3 0 7 1 0  18  (BRONZE)
>>BRA2 = Samuel 2 2 0 7 0 7  18  (BRONZE)
>>BRA3 = Rafael 0 1 0 7 3 0  11  (MH)
>>BRA4 = Larissa1 0 0 7 0 1   9  (MH)
>>BRA5 = Fabio  7 3 0 7 1 1  19  (PRATA)
>>BRA6 = David  2 3 0 7 4 1  17  (BRONZE)
>>
>>Total 1912 042 910  92
>>
>>Tr^es estudantes fizeram 42 pontos, um da China e dois do Vietnam.
>>O pais com o maior total de pontos foi a Bulgaria, significativamente
>>aa frante da China, 2a colocada. A Argentina conquistou uma medalha de
ouro
>>(com 29 pontos) mas ficou 1 ponto atras do Brasil no total de pontos.
>>
>>As proximas IMOs ser~ao na Grecia (2004), Mexico (2005), Eslovenia (2006)
>>e Vietnam (2007).
>>
>>[]s, Nicolau
>
>Resultado geral em 
>http://village.infoweb.ne.jp/~fvgm9250/web/imo2003/contents/news.html
>[]s.
>joao dias
>>==
>>Instruções para entrar na lista, sair da lista e usar a lista em
>>http://www.mat.puc-rio.br/~nicolau/olimp/obm-l.html
>>=
>
>
>=
>Instruções para entrar na lista, sair da lista e usar a lista em
>http://www.mat.puc-rio.br/~nicolau/olimp/obm-l.html
>=
>

[]'s, Yuri
ICQ: 64992515


--
Use o melhor sistema de busca da Internet
Radar UOL - http://www.radaruol.com.br



=
Instruções para entrar na lista, sair da lista e usar a lista em
http://www.mat.puc-rio.br/~nicolau/olimp/obm-l.html
=


[obm-l] Re: [obm-l] geometria

2003-07-24 Thread yurigomes

Oi Rodrigo,
 Seja ABC=B e ACB=C. Então NCP= 90- C/2. Como NP//BC, temos CNP=C. Logo,
NPC= 180- (C+ 90- C/2)= 90- C/2 => CNP isósceles => NP=NC=7 => MN+ MP= 7.

 De modo análogo, BMP= 180- B e MBP= B/2 => BPM= B/2 => BMP isósceles =>
MP= MB= 4. 
 Logo, MN= 7- MP= 7- 4 => MN= 3.
 Ateh mais, 

  Yuri

-- Mensagem original --

>Num triangulo qualquer ABC, traçamos a bissetriz interna de B e a bissetriz
>
>externa de C. Pelo ponto P de concurso das bissetrizes, traçamos uma reta
>
>paralela ao lado BC , encontrando o lado AB no ponto M e AC no ponto N.

>Calcule o segmento MN sabendo que MB= 7 cm e NC= 4 cm
>
>[EMAIL PROTECTED]
>
>_
>MSN Hotmail, o maior webmail do Brasil.  http://www.hotmail.com
>
>=
>Instruções para entrar na lista, sair da lista e usar a lista em
>http://www.mat.puc-rio.br/~nicolau/olimp/obm-l.html
>=
>

[]'s, Yuri
ICQ: 64992515


--
Use o melhor sistema de busca da Internet
Radar UOL - http://www.radaruol.com.br



=
Instruções para entrar na lista, sair da lista e usar a lista em
http://www.mat.puc-rio.br/~nicolau/olimp/obm-l.html
=


[obm-l] Re: [obm-l] geometria 2

2003-07-24 Thread yurigomes
  Sejam a e b os comprimentos dos catetos, I o incentro de C1 e X o ponto
de tangência de C1 com AC. Então o raio de C é igual a AX, e eh esse valor
vale r= p- Hipotenusa= (a+b-Hipotenusa)/2= [a+b- sqr(a^2+b^2)]/2= k/2 -
sqr(a^2+b^2)/2 
  O raio de C2 é a metade da hipotenusa: R= sqr(a^2+b^2)/2. Assim, a soma
dos comprimentos de C1 e C2 é igual a 2pi(R+ r)= pi(k/2 - sqr(a^2+b^2)/2
+sqr(a^2+b^2)/2)= 2pi.k/2= pi.k
 Ateh mais, 
 Yuri

-- Mensagem original --

>Rodrigo Salcedo, eu aqui de novo!!!
>Consideremos um triangulo retangulo que simultaneamente esta circunscrito
>à 
>circunferencia C1 e inscrito à circunferencia C2 . Sabendo-se que a soma
>dos 
>comprimentos dos catetos do triangulo é K  cm, qual sera a soma dos 
>comprimentos destas duas circunferencias?
>
>[EMAIL PROTECTED]
>
>_
>MSN Hotmail, o maior webmail do Brasil.  http://www.hotmail.com
>
>=
>Instruções para entrar na lista, sair da lista e usar a lista em
>http://www.mat.puc-rio.br/~nicolau/olimp/obm-l.html
>=
>

[]'s, Yuri
ICQ: 64992515


--
Use o melhor sistema de busca da Internet
Radar UOL - http://www.radaruol.com.br



=
Instruções para entrar na lista, sair da lista e usar a lista em
http://www.mat.puc-rio.br/~nicolau/olimp/obm-l.html
=


[obm-l] Re: [obm-l] Re: [obm-l] geometria

2003-07-25 Thread yurigomes
 É só trocar. 
 É que na minha figura M e N ficaram acima de A, qdo a posição correta é
abaixo.

-- Mensagem original --

>Ola pessoal,
>
>No enunciado foi dito que MB= 7 cm e NC= 4 cm, mas na resolucao eh dito
que
>
>MB= 4 e NC = 7. Eh assim mesmo ?
>
>
>
>Em uma mensagem de 24/7/2003 23:44:20 Hora padrão leste da Am. Sul, 
>[EMAIL PROTECTED] escreveu:
>
>
>> Oi Rodrigo,
>> Seja ABC=B e ACB=C. Então NCP= 90- C/2. Como NP//BC, temos CNP=C. Logo,
>> NPC= 180- (C+ 90- C/2)= 90- C/2 => CNP isósceles => NP=NC=7 => MN+ MP=
>7.
>> 
>> De modo análogo, BMP= 180- B e MBP= B/2 => BPM= B/2 => BMP isósceles
=>
>> MP= MB= 4. 
>> Logo, MN= 7- MP= 7- 4 => MN= 3.
>> Ateh mais, 
>> 
>>   Yuri
>> 
>> -- Mensagem original --
>> 
>> >Num triangulo qualquer ABC, traçamos a bissetriz interna de B e a bissetriz
>> >
>> >externa de C. Pelo ponto P de concurso das bissetrizes, traçamos uma
reta
>> >
>> >paralela ao lado BC , encontrando o lado AB no ponto M e AC no ponto
N.
>> 
>> >Calcule o segmento MN sabendo que MB= 7 cm e NC= 4 cm
>> >
>> >[EMAIL PROTECTED]
>
>
>

[]'s, Yuri
ICQ: 64992515


--
Use o melhor sistema de busca da Internet
Radar UOL - http://www.radaruol.com.br



=
Instruções para entrar na lista, sair da lista e usar a lista em
http://www.mat.puc-rio.br/~nicolau/olimp/obm-l.html
=


[obm-l] Re: [obm-l] Re: [obm-l] geometria 2

2003-07-25 Thread yurigomes
 Se naum me engano na notação adotada no problema, o valor de AX é sempre
igual a p-a, onde p é o semiperímetro e a= BC, independente do triângulo.
No caso em que o triângulo, os pontos A, X, I e o outro pto de tangência
da circunferência inscrita a ABC formam um quadrado, e assim os lados são
todos iguais, ou seja, IX= AX. Mas IX=r, donde r= AX= p- a. 
  Para ver que vale AX= p-a, faça o seguinte: sejm Y e Z os outros ptos
de tangência com os lados AB e BC, resp.. Então:
   AX=AY=x ; CX= CZ= z e  BZ= BY= y. Logo, 
  c=AB= AY+ BY= x+ y
  a=BC= BZ+ CZ= y+ z
  b=AC= CX+ AX= z+ x
  Logo, vc fica com o sistema:
   x+y=c
   y+z=a
   x+z=b
 As soluções dão x=(b+c-a)/2= p- a, onde p= (a+b+c)/2 

-- Mensagem original --

>Ola pessoal,
>
>Direi minha duvida no corpo da mensagem. Para o Yuri ou quem souber.
>
>
>Em uma mensagem de 24/7/2003 23:53:41 Hora padrão leste da Am. Sul, 
>[EMAIL PROTECTED] escreveu:
>
>
>> Sejam a e b os comprimentos dos catetos, I o incentro de C1 e X o ponto
>> de tangência de C1 com AC. Então o raio de C é igual a AX, e eh esse
valor
>> vale r= p- Hipotenusa= (a+b-Hipotenusa)/2= [a+b- sqr(a^2+b^2)]/2= k/2
-
>> sqr(a^2+b^2)/2 (NAO ENTENDI POR QUE VC FEZ r= p- hipotenusa, A PRINCIPIO
>
>> PENSEI QUE p FOSSE O SEMI-PERIMETRO E ESTA EQUACAO TIVESSE ORIGEM NA
FORMULA
>DE 
>> HERON ( S (AREA) = P*R) ,ONDE R EH O RAIO DA CIRCUNFERENCIA INSCRITA
NO
>
>> TRIANGULO, MAS ACHO QUE ME ENGANEI. O QUE FOI FEITO ?)
>>   O raio de C2 é a metade da hipotenusa: R= sqr(a^2+b^2)/2. Assim, a
soma
>> dos comprimentos de C1 e C2 é igual a 2pi(R+ r)= pi(k/2 - sqr(a^2+b^2)/2
>> +sqr(a^2+b^2)/2)= 2pi.k/2= pi.k
>> Ateh mais, 
>> Yuri
>> 
>> -- Mensagem original --
>> 
>> >Rodrigo Salcedo, eu aqui de novo!!!
>> >Consideremos um triangulo retangulo que simultaneamente esta circunscrito
>> >à 
>> >circunferencia C1 e inscrito à circunferencia C2 . Sabendo-se que a
soma
>> >dos 
>> >comprimentos dos catetos do triangulo é K  cm, qual sera a soma dos

>> >comprimentos destas duas circunferencias?
>> >
>> >[EMAIL PROTECTED]
>> >
>> 
>
>
>

[]'s, Yuri
ICQ: 64992515


--
Use o melhor sistema de busca da Internet
Radar UOL - http://www.radaruol.com.br



=
Instruções para entrar na lista, sair da lista e usar a lista em
http://www.mat.puc-rio.br/~nicolau/olimp/obm-l.html
=


[obm-l] Re: [obm-l] IMC - problema 4

2003-07-30 Thread yurigomes

 Vou dar minha solução: eu considerei AUB=N partição. 
  Sejam a, b tais que 
 a.A= b.B.
  Podemos supor, WLOG, que 1 está em A. Então a está em B, de modo que existe
d em B tal que a=b.d. Temos então que b|a, e ainda  aA=db.A => d.A=B. 
 Nosso problema se restringiu então a acharmos d natural tal que d.A=B,
onde AUB é alguma partição de N. Afirmo que todo d satisfaz tal condição.
De fato, cada natural n é representado de modo único na forma d^a.c, onde
d não divide c. Seja então
  A= {d^a.c ; a é par e d não divide c}
  B= {d^a.c ; a é ímpar e d não divide c}
 Claramente AUB= N e d.A=B. 
 Logo, os pares (a,b) que satisfazem são os que satisfazem mdc(a, b)=min{a,
b}.
  Se virem algum erro, me avisem!!

-- Mensagem original --

>
>4. Determine the set of all pairs (a,b) of positive integers for which
the
>set of positive integers can be decomposed into two sets A and B such that
>a.A = b.B.
>
>Seja N = conjunto dos inteiros positivos.
>
>O enunciado fala em decompor N e não particionar N.
>Pra mim, isso significa que devemos ter A U B = N, mas não necessariamente
>A
>inter B = vazio.
>Com essa interpretacao, eu fiz o seguinte:
>
>Consideremos os pares da forma (a,ka), com k inteiro positivo.
>Nesse caso, basta tomar A = kN e B = N para garantir que teremos:
>A U B = N  e  aA = a(kN) = (ka)N = (ka)B.
>
>De forma analoga, podemos tomar todos os pares da forma (kb,b), com k
>inteiro positivo.
>
>Suponhamos agora que algum par (a,b) satisfaz ao enunciado sem que tenhamos
>a | b ou b | a.
>Isso significa que d < a  e  d < b, onde d = mdc(a,b).
>Podemos escrever a = a1*d e b = b1*d, com mdc(a1,b1) = 1.
>
>Sejam os conjuntos correspondentes A e B tais que A U B = N  e  aA = bB.
>
>aA = bB ==>
>a1*d*A = b1*d*B ==>
>a1*A = b1*B
>
>Podemos supor s.p.d.g. que 1 pertence a A.
>Nesse caso, a1 pertence a a1*A ==>
>a1 pertence a b1*B ==>
>existe m em B tal que a1 = b1*m ==>
>b1 | a1 ==>
>b1 = 1 (ja que mdc(a1,b1) = 1).
>
>Mas, b1 = 1 ==>
>a = a1*d, b = b1*d = d ==>
>b divide a ==>
>contradicao
>
>Logo, se a nao divide b e b nao divide a, entao (a,b) nao satisfaz ao
>enunciado.
>
>Conclusao: os unicos pares ordenados de inteiros positivos que satisfazem
>ao
>enunciado sao aqueles nos quais uma das coordenadas eh um multiplo da outra.
>
>
>Um abraco,
>Claudio. 
>=
>Instruções para entrar na lista, sair da lista e usar a lista em
>http://www.mat.puc-rio.br/~nicolau/olimp/obm-l.html
>=
>

[]'s, Yuri
ICQ: 64992515


--
Use o melhor sistema de busca da Internet
Radar UOL - http://www.radaruol.com.br



=
Instruções para entrar na lista, sair da lista e usar a lista em
http://www.mat.puc-rio.br/~nicolau/olimp/obm-l.html
=


[obm-l] Re: [obm-l] IMC dia 2

2003-07-30 Thread yurigomes
   Consegui o item (a). Tou tentando o (b). Sem alguém puder ajudar..
  

5. (a) Show that for each funtion f:QxQ -> R there exists a fnction g:Q->R
such that f(x,y)<=g(x)+g(y) for all x,y in Q.
(b) Find a function f:RxR -> R for which there is no function g:R->R such
that f(x,y) <= g(x) + g(y) for all x,y in R.
 
  (a) Seja Q= {a_1, a_2,..., a_n,...}. Vamos construir g indutivamente,
seja g(a_1)= f(a_1, a_1)/2. Definimos g(a_2) pondo
g(a_2)= máx{f(a_1, a_2)- g(a_1), f(a_2, a_1)- g(a_1), f(a_2, a_2)/2}
 Então a condição f(x,y) <= g(x)+ g(y) vale qdo x,y pertencem a {a_1, a_2}.
Suponha que já definimos g(a_1),...,g(a_n). 
Defina g(a_(n+1))= máx{f(x,a_(n+1))- g(x); x=a_i, i<= n}U{f(a_(n+1),x)-
g(x); x=a_i, i<= n}U{f(a_(n+1), a_(n+1))/2}. Dessa forma, f(x,y) <= g(x)+
g(y) vale sempre que x,y pertencem a {a_1, a_2,...,a_(n+1)}. Seguindo dessa
forma, obtemos g:Q->R tal que f(x,y) <= g(x)+g(y), para todos x,y em Q.


[]'s, Yuri
ICQ: 64992515


--
Use o melhor sistema de busca da Internet
Radar UOL - http://www.radaruol.com.br



=
Instruções para entrar na lista, sair da lista e usar a lista em
http://www.mat.puc-rio.br/~nicolau/olimp/obm-l.html
=


[obm-l] Re: [obm-l] Re: [obm-l] IMC - problema 4

2003-07-30 Thread yurigomes
 Ops, me esqueci de falar que d>1 (!!) A solução é então (a,b) tais que
a!=b e mdc(a, b)=min{a,b}

-- Mensagem original --

>
> Vou dar minha solução: eu considerei AUB=N partição. 
>  Sejam a, b tais que 
> a.A= b.B.
>  Podemos supor, WLOG, que 1 está em A. Então a está em B, de modo que
existe
>d em B tal que a=b.d. Temos então que b|a, e ainda  aA=db.A => d.A=B. 
> Nosso problema se restringiu então a acharmos d natural tal que d.A=B,
>onde AUB é alguma partição de N. Afirmo que todo d satisfaz tal condição.
>De fato, cada natural n é representado de modo único na forma d^a.c, onde
>d não divide c. Seja então
>  A= {d^a.c ; a é par e d não divide c}
>  B= {d^a.c ; a é ímpar e d não divide c}
> Claramente AUB= N e d.A=B. 
> Logo, os pares (a,b) que satisfazem são os que satisfazem mdc(a, b)=min{a,
>b}.
>  Se virem algum erro, me avisem!!
>
>-- Mensagem original --
>
>>
>>4. Determine the set of all pairs (a,b) of positive integers for which
>the
>>set of positive integers can be decomposed into two sets A and B such
that
>>a.A = b.B.
>>
>>Seja N = conjunto dos inteiros positivos.
>>
>>O enunciado fala em decompor N e não particionar N.
>>Pra mim, isso significa que devemos ter A U B = N, mas não necessariamente
>>A
>>inter B = vazio.
>>Com essa interpretacao, eu fiz o seguinte:
>>
>>Consideremos os pares da forma (a,ka), com k inteiro positivo.
>>Nesse caso, basta tomar A = kN e B = N para garantir que teremos:
>>A U B = N  e  aA = a(kN) = (ka)N = (ka)B.
>>
>>De forma analoga, podemos tomar todos os pares da forma (kb,b), com k
>>inteiro positivo.
>>
>>Suponhamos agora que algum par (a,b) satisfaz ao enunciado sem que tenhamos
>>a | b ou b | a.
>>Isso significa que d < a  e  d < b, onde d = mdc(a,b).
>>Podemos escrever a = a1*d e b = b1*d, com mdc(a1,b1) = 1.
>>
>>Sejam os conjuntos correspondentes A e B tais que A U B = N  e  aA = bB.
>>
>>aA = bB ==>
>>a1*d*A = b1*d*B ==>
>>a1*A = b1*B
>>
>>Podemos supor s.p.d.g. que 1 pertence a A.
>>Nesse caso, a1 pertence a a1*A ==>
>>a1 pertence a b1*B ==>
>>existe m em B tal que a1 = b1*m ==>
>>b1 | a1 ==>
>>b1 = 1 (ja que mdc(a1,b1) = 1).
>>
>>Mas, b1 = 1 ==>
>>a = a1*d, b = b1*d = d ==>
>>b divide a ==>
>>contradicao
>>
>>Logo, se a nao divide b e b nao divide a, entao (a,b) nao satisfaz ao
>>enunciado.
>>
>>Conclusao: os unicos pares ordenados de inteiros positivos que satisfazem
>>ao
>>enunciado sao aqueles nos quais uma das coordenadas eh um multiplo da
outra.
>>
>>
>>Um abraco,
>>Claudio. 
>>=
>>Instruções para entrar na lista, sair da lista e usar a lista em
>>http://www.mat.puc-rio.br/~nicolau/olimp/obm-l.html
>>=
>>
>
>[]'s, Yuri
>ICQ: 64992515
>
>
>--
>Use o melhor sistema de busca da Internet
>Radar UOL - http://www.radaruol.com.br
>
>
>
>=
>Instruções para entrar na lista, sair da lista e usar a lista em
>http://www.mat.puc-rio.br/~nicolau/olimp/obm-l.html
>=
>

[]'s, Yuri
ICQ: 64992515


--
Use o melhor sistema de busca da Internet
Radar UOL - http://www.radaruol.com.br



=
Instruções para entrar na lista, sair da lista e usar a lista em
http://www.mat.puc-rio.br/~nicolau/olimp/obm-l.html
=


[obm-l] Re: [obm-l] IMC - problema 1

2003-07-31 Thread yurigomes
  Oi pessoal, segue abaixo a minha solução do problema 1 do primeiro dia
da IMC. Ah, parabéns aa equipe brasileira!! Foi um ótimo resultado para
uma primeira participação nessa competição!!!
 
 
1)a) Seja a1, a2, ... , an, ... uma sequencia de
numeros reais tais que a1=1 e 
a(n+1)>3/2*an para todo n. Prove que a sequencia
   
a(n)

 (3/2)^(n-1)

tem um limite finito ou tende a infinito

b) Prove que para todo alfa>1 existe uma sequencia
a1,a2, ... ,an, ..., com as
mesmas propriedades , tal que
 

lim  a(n)
n->oo  --- = alfa
   (3/2)^(n-1)

 Sol.:
 Defina b(n) tal que a(n+1)=3/2.a(n).b(n). Então pela condição do problema
temos b(n)>= 1. Além disso,multiplicando essas n-1 primeiras igualdades,
temos:
a(2)= 3/2.a(1).b(1)
.
. => a(n)= (3/2)^(n-1).b(1)...b(n-1) (*) =>
.
a(n)= 3/2.a(n-1).b(n-1)

   a(n)/[(3/2)^(n-1)]= b(1)...b(n-1)
  Como b(i)>1, a sequência c(n-1)= b(1)...b(n-1) é estritamente crescente.
Logo tem um limite finito ou tende ao infinito. 
 
  (b) Continuando a trabalhar com os b(i)´s, faça 
  b(n)= alfa^(2^(-k)). 
   Como alfa >1, temos b(n)>1, e ainda 
 b(1)...b(n-1)= alfa[1-2^(-n)], de modo que lim c(n-1)= alfa. Os
a(n)´s ficam definidos então por (*):
 a(n)= (3/2)^(n-1).alfa[1-2^(-n)].
 
 Ateh mais, 
  Yuri

[]'s, Yuri
ICQ: 64992515


--
Use o melhor sistema de busca da Internet
Radar UOL - http://www.radaruol.com.br



=
Instruções para entrar na lista, sair da lista e usar a lista em
http://www.mat.puc-rio.br/~nicolau/olimp/obm-l.html
=


[obm-l] Re: [obm-l] Paradoxo

2003-07-31 Thread yurigomes
  O que acontece nessa figura é que os dois coeficientes angulares das retas
que são "coladas" não são os mesmos, de modo que fica sobrando um espaço
na colagem, equivalente a uma quadrado de lado um, exatamente a diferença
entre as áreas das duas figuras. 

-- Mensagem original --

>O Biagio Taffarel <[EMAIL PROTECTED]> mandou isto para a lista
>em um arquivo grande demais. Normalmente eu não reenvio mas este paradoxo
>é legal, aí vai a mesma figura em versão mais enxuta.
>
>[]s, N.
>

[]'s, Yuri
ICQ: 64992515


--
Use o melhor sistema de busca da Internet
Radar UOL - http://www.radaruol.com.br



=
Instruções para entrar na lista, sair da lista e usar a lista em
http://www.mat.puc-rio.br/~nicolau/olimp/obm-l.html
=


[obm-l] IMC - problema 2 (2o dia)

2003-08-01 Thread yurigomes
 Segue abaixo a solução do problema 2 do 2o dia. Vou deixar um espaço pra
quem quiser tentar!

 02) Calcule o seguinte limite
   2x
   /
lim|   (sin t)^m/t^n dt  (m,n naturais)
x->0+  /
   x

.
.
.
.
.
.
.
.
.
.
.
.
.
.
.
.
Pronto!  Vamos dividir em três casos:
 (i) m< n-1: pelo teorema do valor médio, existe t em (x, 2x) tal que int(x
ateh 2x) (sent)^m/t^n dt = x.(sent)^m/t^n. Logo, lim(x->0+) int(x ateh 2x)
(sent)^m/t^n dt=
lim(x->0+) x.(sent)^m/t^n=
lim(x->0+)(x/t).(sent/t)^m.[1/t^(n-1-m)].
 Nesse último limite, (sent/t)^m -> 1 (pois sent/t -> 1), x/t é limitado
(pois t em (x,2x) => 1/2 < x/t < 1) e [1/t^(n-1-m)]
-> oo, já que n-1-m > 0. Logo, nesse caso, o limite é oo.
 (ii) m> n-1: seguindo o mesmo raciocínio, temos
lim(x->0+) int(x ateh 2x) (sent)^m/t^n dt=
lim(x->0+) x.(sent)^m/t^n=
lim(x->0+)(x/t).(sent/t)^(n-1).(sent)^(m-n+1)
  De modo análogo, x/t é limitado, (sent/t)^(n-1)-> 1 e
(sent)^(m-n+1)-> 0. Assim, o limite é igual a zero.
 (iii) m=n-1: se fizermos o mesmo raciocínio, concluiremos apenas que o
limite está entre 1/2 e 1, mas não chegamos a nenhuma fórmula fechada. Façamos
o seguinte: temos
int(x ateh 2x) (sent)^m/t^n dt=
int(x ateh 2x) (sent)^m/t^(m+1) dt
 Derivando por partes a expressão (sent/t)^m, concluímos que
  (sen2x/2x)^m - (senx/x)^m=
int(x ateh 2x){[m.(sent)^(m-1).cost]/t^m - [m.(sent)^m]/t^(m+1)
 Passando o limite, o lado esquerdo fica igual a 0. de modo que
lim int(x ateh 2x) (sent)^m]/t^(m+1) dt =
lim int(x ateh 2x)[.sent^(m-1).cost]/t^m dt
 Não é difícil provar que
 lim int(x ateh 2x)[.sent^(m-1).cost]/t^m dt=
 lim int(x ateh 2x)[.sent^(m-1)]/t^m dt
 Isso se deve ao fato de cost->1 qdo t->0. Temos então
 lim int(x ateh 2x) (sent)^m]/t^(m+1) dt =
lim int(x ateh 2x)[.sent^(m-1)]/t^m dt .
  Em particular,
 lim int(x ateh 2x) (sent)^m]/t^(m+1) dt =
lim int(x ateh 2x) (sent)^0]/t^(0+1) dt =
lim int(x ateh 2x) 1/t dt= ln(2x)-lnx= ln2.

 Algumas passagens podem ter ficado obscuras, e eventualmente pode haver
algum erro. Qq coisa, me avisem!
  Ateh mais,
  Yuri
 


[]'s, Yuri
ICQ: 64992515


--
Use o melhor sistema de busca da Internet
Radar UOL - http://www.radaruol.com.br



=
Instruções para entrar na lista, sair da lista e usar a lista em
http://www.mat.puc-rio.br/~nicolau/olimp/obm-l.html
=


[obm-l] Re: [obm-l] Cochilo na aula de algebra

2003-08-09 Thread yurigomes
 Todas as raízes são iguais a 2. De fato, se as raízes são x_1, x_2,...,
x_10, então pelas relações de Girard temos:
  x_1 + x_2 + ... +  x_10= 20
  x_1.x_2...x_10= 1024
 Como as raízes são reais positivas, podemos usar MA >= MG:
(x_1 + x_2 + ... +  x_10)/10 >= (x_1.x_2...x_10)^(1/10) =>
20/10 >= (1024)^(1/10) => 2 >= 2
 Como ocorre a igualdade, devemos ter que todos os x_i´s são iguais, logo
10.x_1=20 => x_1= x_2=...= x_10= 2.
  Ateh mais,  
  Yuri

-- Mensagem original --

>Caros colegas:
>
>Aqui vai um bonitinho:
>
>Um estudante acordou no fim de uma aula de algebra a tempo de ouvir o
>professor falar: "...e vou dar uma dica pra voces: todas as raizes sao
reais
>e positivas."
>
>Quando ele olhou pro quadro-negro, viu uma equacao polinomial de 10o. grau
>que o professor tinha dado como dever de casa e que ele comecou a copiar
>feito um maluco.
>
>Infelizmente, o professor apagou rapidamente o quadro e ele soh teve tempo
>de copiar os dois primeiros termos: x^10 - 20*x^9. Ele tambem reparou que
>o
>termo constante era +1024.
>
>Pergunta: Quais as raizes da equacao?
>
>Um abraco,
>Claudio.
>
>=
>Instruções para entrar na lista, sair da lista e usar a lista em
>http://www.mat.puc-rio.br/~nicolau/olimp/obm-l.html
>=
>

[]'s, Yuri
ICQ: 64992515


--
Use o melhor sistema de busca da Internet
Radar UOL - http://www.radaruol.com.br



=
Instruções para entrar na lista, sair da lista e usar a lista em
http://www.mat.puc-rio.br/~nicolau/olimp/obm-l.html
=


[obm-l] Re: [obm-l] Problemas em Aberto - Algarismos

2003-08-10 Thread yurigomes

 Essa primeira questão pode conte repetições, como por exemplo 33600???

-- Mensagem original --

>Caros colegas:
>
>Aqui vao dois problemas que ainda estao em aberto na lista. O primeiro
foi
>enviado pelo Duda Stabel. O segundo eh da olimpiada iraniana, se nao me
>engano.
>
>1) Determinar o conjunto de números inteiros positivos que satisfazem à
duas
>condições: (i) todo número possui exatamente dois algarismos não-nulos,
>sendo um deles o três(3), (ii) todo número é quadrado perfeito.
>
>2) Prove ou disprove: existe uma potencia de 2 tal que ao se permutar os
>algarismos de sua representacao decimal obtem-se uma outra potencia de
2.
>
>Esse segundo tem uma solucao aparentemente simples, mas esta solucao exclui
>o caso de potencias de 2 com algarismos "0" internos (ou seja, numeros
do
>tipo "abcdefg").
>
>Um abraco,
>Claudio.
>
>
>

[]'s, Yuri
ICQ: 64992515


--
Use o melhor sistema de busca da Internet
Radar UOL - http://www.radaruol.com.br



=
Instruções para entrar na lista, sair da lista e usar a lista em
http://www.mat.puc-rio.br/~nicolau/olimp/obm-l.html
=


[obm-l] Re: [obm-l] Re: [obm-l] Cochilo na aula de algebra

2003-08-11 Thread yurigomes

  Oi Henrique, 
 A motivação disso foi o enunciado dizer que todas as raízes são reais e
positivas. Nada melhor do que média nesse caso!!
 Se não houvesse igualdade, nada garantiria que as raízes fossem todas iguais
a 2. De fato, poderiam haver várias possibilidades para o conjunto das dez
raízes. 
 Ateh mais,  
 Yuri
-- Mensagem original --

>> Todas as raízes são iguais a 2. De fato, se as raízes são x_1, x_2,...,
>> x_10, então pelas relações de Girard temos:
>>   x_1 + x_2 + ... +  x_10= 20
>>   x_1.x_2...x_10= 1024
>>  Como as raízes são reais positivas, podemos usar MA >= MG:
>> (x_1 + x_2 + ... +  x_10)/10 >= (x_1.x_2...x_10)^(1/10) =>
>> 20/10 >= (1024)^(1/10) => 2 >= 2
>>  Como ocorre a igualdade, devemos ter que todos os x_i´s são iguais,
logo
>> 10.x_1=20 => x_1= x_2=...= x_10= 2.
>
>Yuri,
>Tá certo, a solução é muito boa... Mas como você pensou em usar MA e MG?
>Já
>conhecia o problema (ou algum parecido?)
>E se, no caso, as médias fossem diferentes? Não daria pra sair daí?
>Desculpe pela dúvida um tanto quanto "idiota", mas não custa perguntar...
>
>Grato,
>Henrique.
>
>=
>Instruções para entrar na lista, sair da lista e usar a lista em
>http://www.mat.puc-rio.br/~nicolau/olimp/obm-l.html
>=
>

[]'s, Yuri
ICQ: 64992515


--
Use o melhor sistema de busca da Internet
Radar UOL - http://www.radaruol.com.br



=
Instruções para entrar na lista, sair da lista e usar a lista em
http://www.mat.puc-rio.br/~nicolau/olimp/obm-l.html
=


[obm-l] Re: [obm-l] Problemas em Aberto - Algarismos

2003-08-14 Thread yurigomes
 Esse segundo problema caiu na OBM 2000, numa versão mais fácil.
 Acho que foi essa versão a que vc resolveu, jah que ele dizia que as duas
potências têm que ter o mesmo número de algarismos, de modo que os zeros
não modificavam a quantidade de algarismos.
 Ateh mais, 
 Yuri
  
-- Mensagem original --

>Caros colegas:
>
>Aqui vao dois problemas que ainda estao em aberto na lista. O primeiro
foi
>enviado pelo Duda Stabel. O segundo eh da olimpiada iraniana, se nao me
>engano.
>
>1) Determinar o conjunto de números inteiros positivos que satisfazem à
duas
>condições: (i) todo número possui exatamente dois algarismos não-nulos,
>sendo um deles o três(3), (ii) todo número é quadrado perfeito.
>
>2) Prove ou disprove: existe uma potencia de 2 tal que ao se permutar os
>algarismos de sua representacao decimal obtem-se uma outra potencia de
2.
>
>Esse segundo tem uma solucao aparentemente simples, mas esta solucao exclui
>o caso de potencias de 2 com algarismos "0" internos (ou seja, numeros
do
>tipo "abcdefg").
>
>Um abraco,
>Claudio.
>
>
>

[]'s, Yuri
ICQ: 64992515


--
Use o melhor sistema de busca da Internet
Radar UOL - http://www.radaruol.com.br



=
Instruções para entrar na lista, sair da lista e usar a lista em
http://www.mat.puc-rio.br/~nicolau/olimp/obm-l.html
=


[obm-l] Re: [obm-l] Teoria dos Números

2003-08-16 Thread yurigomes

Oi Claudio,

Eu não entendi pq vc considerou polinômios para provar a última passagem,
jah que a está fixo. Ou seja, vc tem que 
a^n - 1 divide a^Phi(a^n - 1) - 1 
 e não que x^n-1 divide x^Phi(a^n - 1) - 1 para todo x.
   Se eu tiver falado alguma besteira, me avisem!
  Ateh mais, 
 Yuri
-- Mensagem original --

>on 16.08.03 05:54, Eduardo Casagrande Stabel at [EMAIL PROTECTED] wrote:
>
>> Olá pessoal!
>> 
>> Prove que se n > 1 e a > 0 são inteiros então n | PHY(a^n - 1).
>> 
>> PHY é a função de Euler.
>> 
>> Abraço,
>> Duda.
>> 
>
>Oi, Duda:
>
>Eh claro que mdc(a,a^n - 1) = 1
> 
>Entao, pelo teorema de Euler, teremos:
>a^Phi(a^n - 1) == 1 (mod a^n - 1) ==>
>
>a^n - 1 divide a^Phi(a^n - 1) - 1 ==>
>
>n divide Phi(a^n - 1)
>
>***
>
>Essa ultima passagem pode ser vista da seguinte forma:
>
>Sejam x^n - 1 e x^n - 1 polinomios (portanto m, n inteiros)
>
>x^n - 1 divide x^m - 1 mas n nao divide m ==>
>
>m = qn + r com 0 < r <= n-1 ==>
>
>x^m - 1 = x^(qn + r) - 1 = x^(qn)*x^r - x^r + x^r - 1 =
>= x^r(x^(qn) - 1) + x^r - 1 ==>
>
>x^n - 1 divide x^r - 1 com 0 < r < n ==>
>
>contradicao.
>
>
>Um abraco,
>Claudio.
>
>=
>Instruções para entrar na lista, sair da lista e usar a lista em
>http://www.mat.puc-rio.br/~nicolau/olimp/obm-l.html
>=
>

[]'s, Yuri
ICQ: 64992515


--
Use o melhor sistema de busca da Internet
Radar UOL - http://www.radaruol.com.br



=
Instruções para entrar na lista, sair da lista e usar a lista em
http://www.mat.puc-rio.br/~nicolau/olimp/obm-l.html
=


[obm-l] Re: [obm-l] Re: [obm-l] Teoria dos Números

2003-08-16 Thread yurigomes
 Exatamente! Vejam a minha msg anterior...

-- Mensagem original --

>Oi, Yuri:
>
>O que eu provei foi o seguinte:
>m divide n <==> p(x) = x^m - 1 divide q(x) = x^n - 1
>(na verdade, eu provei soh a volta, mas a ida eh imediata)
>
>Em particular, com um inteiro a fixo:
>m divide n <==> p(a) divide q(a).
>
>Ou seja, eu provei um resultado mais geral do que eu realmente precisava.
>
>Compare com o seguinte:
>2^2 + 1 divide 2^4 + 4 mas nao eh verdade que f(x) = x^2 + 1 divide g(x)
>=
>x^4 + 4, pois x^4 + 4 = (x^2 + 1)*(x^2 - 1) + 5
>
>Um abraco,
>Claudio.
> 
>on 16.08.03 12:55, [EMAIL PROTECTED] at [EMAIL PROTECTED]
>wrote:
>
>> 
>> Oi Claudio,
>> 
>> Eu não entendi pq vc considerou polinômios para provar a última passagem,
>> jah que a está fixo. Ou seja, vc tem que
>> a^n - 1 divide a^Phi(a^n - 1) - 1
>> e não que x^n-1 divide x^Phi(a^n - 1) - 1 para todo x.
>> Se eu tiver falado alguma besteira, me avisem!
>> Ateh mais, 
>> Yuri
>> -- Mensagem original --
>> 
>>> on 16.08.03 05:54, Eduardo Casagrande Stabel at [EMAIL PROTECTED]
wrote:
>>> 
 Olá pessoal!
 
 Prove que se n > 1 e a > 0 são inteiros então n | PHY(a^n - 1).
 
 PHY é a função de Euler.
 
 Abraço,
 Duda.
 
>>> 
>>> Oi, Duda:
>>> 
>>> Eh claro que mdc(a,a^n - 1) = 1
>>> 
>>> Entao, pelo teorema de Euler, teremos:
>>> a^Phi(a^n - 1) == 1 (mod a^n - 1) ==>
>>> 
>>> a^n - 1 divide a^Phi(a^n - 1) - 1 ==>
>>> 
>>> n divide Phi(a^n - 1)
>>> 
>>> ***
>>> 
>>> Essa ultima passagem pode ser vista da seguinte forma:
>>> 
>>> Sejam x^n - 1 e x^n - 1 polinomios (portanto m, n inteiros)
>>> 
>>> x^n - 1 divide x^m - 1 mas n nao divide m ==>
>>> 
>>> m = qn + r com 0 < r <= n-1 ==>
>>> 
>>> x^m - 1 = x^(qn + r) - 1 = x^(qn)*x^r - x^r + x^r - 1 =
>>> = x^r(x^(qn) - 1) + x^r - 1 ==>
>>> 
>>> x^n - 1 divide x^r - 1 com 0 < r < n ==>
>>> 
>>> contradicao.
>>> 
>>> 
>>> Um abraco,
>>> Claudio.
>>> 
>>> =
>>> Instruções para entrar na lista, sair da lista e usar a lista em
>>> http://www.mat.puc-rio.br/~nicolau/olimp/obm-l.html
>>> =
>>> 
>> 
>> []'s, Yuri
>> ICQ: 64992515
>> 
>> 
>> --
>> Use o melhor sistema de busca da Internet
>> Radar UOL - http://www.radaruol.com.br
>> 
>> 
>> 
>> =
>> Instruções para entrar na lista, sair da lista e usar a lista em
>> http://www.mat.puc-rio.br/~nicolau/olimp/obm-l.html
>> =
>> 
>
>=
>Instruções para entrar na lista, sair da lista e usar a lista em
>http://www.mat.puc-rio.br/~nicolau/olimp/obm-l.html
>=
>

[]'s, Yuri
ICQ: 64992515


--
Use o melhor sistema de busca da Internet
Radar UOL - http://www.radaruol.com.br



=
Instruções para entrar na lista, sair da lista e usar a lista em
http://www.mat.puc-rio.br/~nicolau/olimp/obm-l.html
=


[obm-l] Re: [obm-l] Re: [obm-l] Teoria dos Números

2003-08-16 Thread yurigomes

 Opa! Na verdade vale uma coisa mais geral!
 a^n- 1 divide a^m- 1 <=> n divide m. 
 Dessa forma, tirei a minha dúvida. Além disso, a prova do Cláudio prova
também a afirmação acima.
 Ateh mais, 
 Yuri
-- Mensagem original --

>
>Oi Claudio,
>
>Eu não entendi pq vc considerou polinômios para provar a última passagem,
>jah que a está fixo. Ou seja, vc tem que 
>a^n - 1 divide a^Phi(a^n - 1) - 1 
> e não que x^n-1 divide x^Phi(a^n - 1) - 1 para todo x.
>   Se eu tiver falado alguma besteira, me avisem!
>  Ateh mais, 
> Yuri
>-- Mensagem original --
>
>>on 16.08.03 05:54, Eduardo Casagrande Stabel at [EMAIL PROTECTED] wrote:
>>
>>> Olá pessoal!
>>> 
>>> Prove que se n > 1 e a > 0 são inteiros então n | PHY(a^n - 1).
>>> 
>>> PHY é a função de Euler.
>>> 
>>> Abraço,
>>> Duda.
>>> 
>>
>>Oi, Duda:
>>
>>Eh claro que mdc(a,a^n - 1) = 1
>> 
>>Entao, pelo teorema de Euler, teremos:
>>a^Phi(a^n - 1) == 1 (mod a^n - 1) ==>
>>
>>a^n - 1 divide a^Phi(a^n - 1) - 1 ==>
>>
>>n divide Phi(a^n - 1)
>>
>>***
>>
>>Essa ultima passagem pode ser vista da seguinte forma:
>>
>>Sejam x^n - 1 e x^n - 1 polinomios (portanto m, n inteiros)
>>
>>x^n - 1 divide x^m - 1 mas n nao divide m ==>
>>
>>m = qn + r com 0 < r <= n-1 ==>
>>
>>x^m - 1 = x^(qn + r) - 1 = x^(qn)*x^r - x^r + x^r - 1 =
>>= x^r(x^(qn) - 1) + x^r - 1 ==>
>>
>>x^n - 1 divide x^r - 1 com 0 < r < n ==>
>>
>>contradicao.
>>
>>
>>Um abraco,
>>Claudio.
>>
>>=
>>Instruções para entrar na lista, sair da lista e usar a lista em
>>http://www.mat.puc-rio.br/~nicolau/olimp/obm-l.html
>>=
>>
>
>[]'s, Yuri
>ICQ: 64992515
>
>
>--
>Use o melhor sistema de busca da Internet
>Radar UOL - http://www.radaruol.com.br
>
>
>
>=
>Instruções para entrar na lista, sair da lista e usar a lista em
>http://www.mat.puc-rio.br/~nicolau/olimp/obm-l.html
>=
>

[]'s, Yuri
ICQ: 64992515


--
Use o melhor sistema de busca da Internet
Radar UOL - http://www.radaruol.com.br



=
Instruções para entrar na lista, sair da lista e usar a lista em
http://www.mat.puc-rio.br/~nicolau/olimp/obm-l.html
=


[obm-l] Re: imo

2003-08-17 Thread yurigomes
 Se não me engano, o problema C5 é do Gugu. 
 Ele caiu uma semana atrás no teste pra Ibero desse ano. 
 Ateh mais, 
 Yuri

[]'s, Yuri
ICQ: 64992515


--
Use o melhor sistema de busca da Internet
Radar UOL - http://www.radaruol.com.br



=
Instruções para entrar na lista, sair da lista e usar a lista em
http://www.mat.puc-rio.br/~nicolau/olimp/obm-l.html
=


[no subject]

2003-08-17 Thread yurigomes
 Se não me engano, o problema C5 é do Gugu. 
 Ele caiu uma semana atrás no teste pra Ibero desse ano. 
 Ateh mais, 
 Yuri

[]'s, Yuri
ICQ: 64992515


--
Use o melhor sistema de busca da Internet
Radar UOL - http://www.radaruol.com.br



=
Instruções para entrar na lista, sair da lista e usar a lista em
http://www.mat.puc-rio.br/~nicolau/olimp/obm-l.html
=


Problema de Análise

2001-02-28 Thread yurigomes

 
   Abaixo vai um problema de Análise bem interessante:

   Dada f:[a,b)->R, suponha que f é derivável em (a,b) e tenha derivada
limitada. Prove então que f é limitada.

   Aguardo a solução.
[]'s, Yuri



___

http://www.zipmail.com.br O e-mail que vai aonde você está.






Re: Problemas ....

2001-06-24 Thread yurigomes



-- Mensagem original --

>Caros colegas
>
>As medidas dos lados de um triângulo são x^2 + x + 1, 2x + 1 e x^2 - 1.
>Determine o maior lado e prove que o triângulo tem um ângulo de 120°.
>
>Um abraço
>  YCM!


  Vejamos... Como x^2 - 1 é lado, temos x > 1. Daí
  x^2 + x + 1 > 2x + 1  sss  x^2 > x sss x > 1 (OK)   e obviamente,
x^2 + x + 1 > x^2 - 1.  Logo, x^2 + x + 1 é o maior lado. Agora, por lei
dos cossenos, vem que 
(x^2 + x + 1)^2 = (2x + 1)^2 + (x^2 - 1)^2 - 2(2x+1)(x^2 - 1)cosa
 x^2 + 2x^3 = 2x + 1 - 2(2x+1)(x^2 - 1)cosa
 (2x+1)(x^2 - 1)= - 2(2x+1)(x^2 - 1)cosa
   cosa = -1/2 => a = 120º
 

[]'s, Yuri
ICQ: 64992515


___

http://www.zipmail.com.br O e-mail que vai aonde você está.






Re: Problema-Seleção

2001-07-06 Thread yurigomes

Uma solução simples vai abaixo: 
 Suponha x inteiro e Q(x)=0. Então P(x)P(x^2)P(x^3)P(x^4) = -1.
Será que isso pode acontecer?? Não!!! De fato, devemos ter   P(x^k)
pertencente a {1,-1}, 1<=k<=4 ( pois P tem coeficientes inteiros). Daí,
como x congruente a x^3 e x^2 congrunte a x^4 mod 3 , temos P(x) congruente
a P(x^3) e P(x^2) congruente a P(x^4) mod 3. Logo, visto que -1 é incongruente
a 1 mod 3, temos 
P(x)=P(x^3) e P(x^2)=P(x^4) e daí P(x)P(x^3)=P(x^2)P(x^4)=1, concluindo
que P(x)P(x^2)P(x^3)P(x^4)=1, absurdo!!! 


-- Mensagem original --

>Seja P(x) um polinômio de coeficientes inteiros e seja Q(x), tal que :
>   Q(x) = P(x)*P(x^2)*P(x^3)*P(x^4) + 1. Mostre que Q(x) não possui raízes
>inteiras.
>
>Pô, eu consegui mostrar que se Q(x) possuísse raízes inteiras, só poderiam
>ser 2 ou -2, mas não consegui mostrar que essas não podem ser . Se
alguém
>quiser, mando o que fiz...
>
>¡ Villard !
>

[]'s, Yuri
ICQ: 64992515


___

http://www.zipmail.com.br O e-mail que vai aonde você está.






Afinal, como foi???

2001-07-10 Thread yurigomes

Ei, como eh que vcs foram realmente Eu tou acabando de ver q tu fez
uma questão toda. Naum tou entendendo como eh que foram as pontuações. Tu
jah eh menção né???



[]'s, Yuri
ICQ: 64992515


___

http://www.zipmail.com.br O e-mail que vai aonde você está.






Correção

2001-07-10 Thread yurigomes

 O email "Afinal, como foi?" não era endereçado para a lista.



[]'s, Yuri
ICQ: 64992515


___

http://www.zipmail.com.br O e-mail que vai aonde você está.






Problema interessante

2001-08-12 Thread yurigomes

  Alguém poderia me ajudar no problema abaixo. Ainda naum saí do canto!
  
  Sejam a e b inteiros não-negativos tais que ab>=c^2, onde c é inteiro.
Prove que existe um número n e inteiros x1, x2,...,xn, y1, y2,..., yn tais
que
  x1^2 + x2^2 + ... + xn^2= a , y1^2 + y2^2 + ... + yn^2 = b 
 e x1y1 + x2y2 + ... + xnyn = c

  Parece a recíproca da desigualdade de Cauchy.
PS: xi indica "x índice i"


[]'s, Yuri
ICQ: 64992515


___

http://www.zipmail.com.br O e-mail que vai aonde você está.






Re: Inglaterra-00-Plana

2001-08-25 Thread yurigomes

 Uma outra solução para esta questão é a seguinte: 
  PMN = NPQ e NMQ = NPQ ( ângulos segmentos ). Daí,
  RNQ = NPQ + NQP ( ângulo externo ) => RNQ = PMQ
  Mas RMQ compreende o mesmo arco que RNQ => MQ é bissetriz PMR

 



[]'s, Yuri
ICQ: 64992515


___

http://www.zipmail.com.br O e-mail que vai aonde você está.






Re: Quadrados perfeitos...

2001-10-27 Thread yurigomes

  Basta ver que os quadrados perfeitos só podem acabar em 1,4,5,6 ou 9.
Daih, a resposta eh letra e).


[]'s, Yuri
ICQ: 64992515


___

http://www.zipmail.com.br O e-mail que vai aonde você está.






Re: Função

2001-10-29 Thread yurigomes

 Pensei um pouquinho, e tudo que consegui eh que f(x)= -c/x, x/=0, e f(x)=0
satisfaz, para c real. Serah que tem outra??



[]'s, Yuri
ICQ: 64992515


___

http://www.zipmail.com.br O e-mail que vai aonde você está.






Ternas (x,y,z)

2001-11-21 Thread yurigomes

 Alguém pode resolver ou dar dicas para a seguinte questão???

 Dado um inteiro positivo n, achar todas as ternas (x,y,z) de números reais
tais que
 y*x^n + z*y^n + x*z^n = x*y^n + y*z^n + z*x^n

[]'s, Yuri
ICQ: 64992515


--
Use o melhor sistema de busca da Internet
Radar UOL - http://www.radaruol.com.br






Re: 4 Questoes

2001-11-26 Thread yurigomes

>4) Seja um paralelogramo ABCD. Traça-se uma reta que passa por D e corta
>o lado BC no ponto P e o prolongamento do lado AB no ponto Q. Se a área
do
>triângulo DAC vale 8 e a área do quadrilátero ABCD vale 29, quanto vale
a
>área do triângulo CPQ?

Se a área de DAC=8, então a de ABCD será 16, e não 29. 
Não será a área de ACPQ=29??

[]'s, Yuri
ICQ: 64992515


--
Use o melhor sistema de busca da Internet
Radar UOL - http://www.radaruol.com.br






Re: Um quadrado repartido

2001-11-28 Thread yurigomes

  
  | /|
  |   x/ |1/2
  |  x/  |
  |--|
  |   \  |
  |\ |
  | \|
  
 Espero que a figura satisfaça. Faça Pitágoras no triângulo retângulo da
direita acima.Daí,
 x^2= (1/2)^2 + (1-x)^2 ... x=5/8
Aih a área fica fácil






-- Mensagem original --

>Olá. Alguem pode me ajudar a encontrar a área do triângulo formado
>interno ao quadrado na figura anexa?
>E quanto à medida das semi-retas que cortam o quadrado (as bases
>menores dos trapézios formados, que sao iguais aos lados do triângulo)?
>
>Agradeço a ajuda,
>
> Ricardo Miranda M
>[EMAIL PROTECTED]


[]'s, Yuri
ICQ: 64992515


--
Use o melhor sistema de busca da Internet
Radar UOL - http://www.radaruol.com.br






Re: Pergunta intrigante

2001-11-29 Thread yurigomes

 Para n=3, existe um modo fácil de ver a desigualdade. Basta ver que
  (x + y + z)(x^2+ y^2+ z^2 - xy- yz- zx)>= 0
 Provando a média para dois números, a segunda expressão em parênteses é
>= 0, cqd 



-- Mensagem original --

>Há pouco tempo um aluno me perguntou sobre uma questao do IME 2001,
que
>pedia para demonstrar que (x + y + z)/3 >= 3r(xyz), x>0, y>0, z>0 onde
3r
>está representando "raiz cúbica de" e >= o sinal de "maior ou igual a"
>
>Nós já havíamos trabalhado por alto a desigualdade das médias, daí
ele
>me fez a pergunta que eu nao soube responder:
>
>"Ora, sabemos que a média aritmética de n termos é maior ou igual à
média
>geométrica destes termos. Como vale para n, vale para 3. Resolvido o problema?"
>
>Minha opiniao PARTICULAR é q nao...
>
>É óbvio que eu nao defendo a teoria de que pra usar "Pitágoras" em
uma
>prova temos de antes demonstrá-lo...
>
>Mas também acho que deve haver bom senso na resolucao de uma prova.
O
>que vocês da lista têm a dizer?
>
>Eu resolveria a questao da seguinte maneira:
>
>Seja nr(x) a raiz de índice n do número x.
>
>1) Primeiro provemos que (x+y)/2 >= 2r(xy)  -->  (x+y) >= 2* 2r(xy) -->

>(x+y)^2 >= 4xy -->
>
>(x-y)^2 >= 0, que é sempre verdadeiro.
>
>Assim, analogamente (z+w)/2 >= 2r(zw) e   (c+d)/2 >= 2r(cd)
>
>Seja (x+y+z+w)/4 = a.
>
>a = [(x+y)/2 + (z+w)/2]/2  >=  [2r(xy) + 2r(zw)]/2
>
>Se c = 2r(xy) e  d = 2r(zw), vem:
>
>a >= (c+d)/2 >= 2r(cd) = 2r[2r(xy) * 2r(zw)] = 4r(xyzw)
>
>Fazendo w = (x+y+z)/3, vem:
>
>a = [x+y+z + (x+y+z)/3 ]/4 = (x + y + z)/3 = w
>
>Como a >= 4r(xyzw), entao:
>
>w >= 4r(xyzw)   -->   w^4 >= xyzw  -->  w^3 >= xyz
>
>Ou:
>
>(x+y+z)/3 >= 3r(xyz),   c.q.d.
>
>
>
>
>

[]'s, Yuri
ICQ: 64992515


--
Use o melhor sistema de busca da Internet
Radar UOL - http://www.radaruol.com.br






Re: 3 problemas - urgente

2001-10-14 Thread yurigomes

 Lattice são os pontos de coordenadas inteiras.

[]'s, Yuri
ICQ: 64992515


___

http://www.zipmail.com.br O e-mail que vai aonde você está.






Re: Problemas

2001-10-14 Thread yurigomes


Para a questão 3, tente provar que o ponto de interseção de HM com a circunferência
circunscrita a ABC pertencente ao arco BC é diametralmente oposto a A.
-- Mensagem original --

>
>
>Olá pessoal,
>Olhem só estes problemas:
>1.Quantos triangulos diferentes ,de lados inteiros,podem ser construidos
>de 
>modo que o(s) lado(s) maior(es) tenha(m) 5cm de comprimento? E 6 cm? E
n
>cm? 
>Em cada caso, quantos são isósceles?
>
>2.AEDC e BCFG são quadrados sobre os catetos do triang. retangulo 
>ABC.Traçam-se as retas EB e AG.Prove que a área do triang. AHB é igual
 à
>
>área do quadrilátero CIHJ.
>
>3.Sejam H o ortocentro to triang. ABC, não retangulo,e M o pnt médio do
lado
>
>BC.A circunferencia de diâmetro AM encontra a circunferencia circunscrita
>ao 
>ABC em um segundo ponto P.Mostre que P,H e M são colineares.
>
>  Obrigada!
>  FÊ
>
>_
>Chegou o novo MSN Explorer. Instale já. É gratuito! 
>http://explorer.msn.com.br
>
>

[]'s, Yuri
ICQ: 64992515


___

http://www.zipmail.com.br O e-mail que vai aonde você está.






Divisibilidade num polinômio

2001-12-13 Thread yurigomes

 Oi,
  Alguém sabe mostrar que existe a inteiro tal que 3^2000|P(a), onde P(x)=
x^3+ 153x^2- 111x+ 38???


[]'s, Yuri
ICQ: 64992515


--
Use o melhor sistema de busca da Internet
Radar UOL - http://www.radaruol.com.br






Equação

2001-12-28 Thread yurigomes

 Olah a todos.
 Alguém consegue, por favor, resolver o problema abaixo!!!
 Ache todos x, y inteiros tais que
  2x^4+ 1= y^2



[]'s, Yuri
ICQ: 64992515


--
Use o melhor sistema de busca da Internet
Radar UOL - http://www.radaruol.com.br






[obm-l] Re: [obm-l] Russas

2002-04-20 Thread yurigomes

 Para o problema 1 existe uma solução que prova ainda que 3(x+y)+1 e 4(x+y)+1
são quadrados perfeitos.
 Veja que 
  3x^2+ x= 4y^2+ y <=>  3(x^2- y^2) + (x-y)= y^2 <=>
 <=> (x-y)(3x+3y+1)=y^2
 Seja d=mdc(x-y, 3x+3y+1). Suponha d>1. Então existe p primo ; p|d. Então
p|y^2 -> p|y -> p|(x-y) + y= x  ->  p|(x+y) ->
p| 3(x+y) + 1 - 3(x+y) = 1, absurdo. Logo d=1 -> x-y e 3(x+y)+1 são ambos
quadrados.  
  Se, na equação original, somarmos x^2 e fizermos uma fatoração semelhante,
concluímos que 4(x+y)+1 é quadrado perfeito.

-- Mensagem original --

>  Olá pessoal,
>Olhem estas questões:
>1. Para os inteiros positivos x e y é verdadeira a igualdade : 3x^2 
>+x=4y^2+y. Mostre que x-y é um quadrado perfeito.
>
>2.Seja ABC um triangulo retangulo de hipotenusa AC .Sabendo que sobre o
lado
>
>BC existem pnts  D e E tais que BÂD=DÂE=EÂC  e EC=2BD . Determineos angulos
>
>do triangulo.
>
>3.Eliminando-se o 2000º algarismo an expansão decimal da fração 1/p,p 
>primo>5,  obtemos a fração a/b; mostre q p|b.
>
>  Se alguém puder me dar uma luz eu agradeço!
>  []´s
>   H!
>
>
>
>_
>MSN Photos is the easiest way to share and print your photos: 
>http://photos.msn.com/support/worldwide.aspx
>
>=
>Instruções para entrar na lista, sair da lista e usar a lista em
>http://www.mat.puc-rio.br/~nicolau/olimp/obm-l.html
>O administrador desta lista é <[EMAIL PROTECTED]>
>=
>

[]'s, Yuri
ICQ: 64992515


--
Use o melhor sistema de busca da Internet
Radar UOL - http://www.radaruol.com.br



=
Instruções para entrar na lista, sair da lista e usar a lista em
http://www.mat.puc-rio.br/~nicolau/olimp/obm-l.html
O administrador desta lista é <[EMAIL PROTECTED]>
=



[obm-l] Re: [obm-l] alguém sabe?

2002-04-20 Thread yurigomes


Na verdade, é possível provar que 
  {x>0/ x^x^x^x... converge}= [e^(-e), e^(1/e)]
   
-- Mensagem original --

>Olá Rui,
>  Meu amigo Artur me apresentou esse problema na
>semana passada:
>  Para x > e^(1/e), temos x=e^(1/e+y), onde y > 0
>logo x^x = e^((1/e+y)*e^(1/e+y)) > e^(e^(1/e+y-1)+y)
>, pois e^(1/e+y) > 1. E como e^x > 1 + x para todo x,
>temos e^(e^(1/e+y-1)+y) > e^(1/e+2y). Por inducao se
>prova que:
>  x^x^x^...^x > e^(1/e+n*y)
>n vezes
>
>  Logo a sequencia diverge, eh claro. para x =
>e^(1/e),
>temos: Utilizando a desigualdade e^x <= 1 + (e-1)x,
>quando x <=1 temos.
>  e^(1/e) <= 1 + (e-1)/e.
>  (e^(1/e))^(e^(1/e)) <= e^(1/e*(1+(e-1)/e)) e como:
>1/e + (e-1)/(e^2) < 1 temos e^(1/e*(1+(e-1)/e)) <=
>1+(e-1)/e + ((e-1)/e)^2. Por inducao concluimos que
>x^x^x^...^x <= 1 + (e-1)/e + ((e-1)/e)^2
>+...+((e-1)/e))^n <= e, para todo n. Logo x^
>converge e sabemos que converge para e.
>  Mandei um e-mail inutil, desculpe!
>  Abracos,
>  Humberto Silva Naves
>
> --- Rui Viana <[EMAIL PROTECTED]> escreveu: >
>Olá a todos da lista,
>> Outro dia um amigo meu me apresentou o seguinte
>> problema :
>> Qual a solução para a equação x^x^x^x...=2 ?
>> Bom, a principio x^x^x...=2 => x^2 = 2 => x =
>> 2^(1/2)
>> Mas a equação x^x^x...=4 teria então a solução x =
>> 4^(1/4) = 2^(1/2) ???
>> Então agente fez um teste e descobriu que
>> (2^(1/2))^(2^(1/2))... converge 
>> para 2 e não para 4 (não provamos isso)
>> Daí agente decidiu tentar :
>> Já que seguindo essa linha de raciocinio x^x^x...=n
>> tem solução x=n^(1/n), 
>> faça f(n) = n^(1/n).
>> Eu queria saber para que valor g(n) =
>> f(n)^f(n)^f(n)... converge ??
>> Parece que pra 0> 1/e> n>e g(n) é convexa e converge para algum valor.
>> Alguém consegue provar qualquer desses fatos sobre
>> g(n) ?
>> []'s,
>> Rui L Viana F
>> [EMAIL PROTECTED]
>> 
>>
>_
>> MSN Photos is the easiest way to share and print
>> your photos: 
>> http://photos.msn.com/support/worldwide.aspx
>> 
>>
>=
>> Instruções para entrar na lista, sair da lista e
>> usar a lista em
>> http://www.mat.puc-rio.br/~nicolau/olimp/obm-l.html
>> O administrador desta lista é
>> <[EMAIL PROTECTED]>
>>
>=
>
>
>___
>Yahoo! Empregos
>O trabalho dos seus sonhos pode estar aqui. Cadastre-se hoje mesmo no Yahoo!
>Empregos e tenha acesso a milhares de vagas abertas!
>http://br.empregos.yahoo.com/
>=
>Instruções para entrar na lista, sair da lista e usar a lista em
>http://www.mat.puc-rio.br/~nicolau/olimp/obm-l.html
>O administrador desta lista é <[EMAIL PROTECTED]>
>=
>

[]'s, Yuri
ICQ: 64992515


--
Use o melhor sistema de busca da Internet
Radar UOL - http://www.radaruol.com.br



=
Instruções para entrar na lista, sair da lista e usar a lista em
http://www.mat.puc-rio.br/~nicolau/olimp/obm-l.html
O administrador desta lista é <[EMAIL PROTECTED]>
=



[obm-l] Re: [obm-l] t. dos nºs

2002-06-11 Thread yurigomes

 Oi Salvador,
 Vc confundiu o problema. A equação é  
 p^2= a^2 = b^2 e não p= a^2 = b^2
 De fato, no livro Introdução à Teoria dos Números, capítulo 7, existe um
teorema que diz que um inteiro n é representado como soma de dois quadrados
se e somente se os expoentes dos primos congruentes a 3 mod 4 que dividem
n são pares. Logo, p^2 pode ser representado dessa forma

 Ateh mais
-- Mensagem original --

>
>
>O primeiro problema so pode ter solucao se p=4n+1. 
>
>Para ver isso, observe que a deve ser par e b impar. Logo a^2+b^2 e da
>forma: 4c^2+4d^2+4c+1, que e da forma 4n+1.
>
>De fato todo primo da forma 4n+1 se escreve de um unico jeito como a soma
>de 2 quadrados. Tem um livro chamado "100 great elementary problems: Their
>history and solutions" Heinrich Dorrie, que tem essa prova e muitas outras
>bacanas. Alias esse livro apresenta as "melhores" provas de cada
>problema. E da Dover e nao e dificil de achar.
>
>
>Abraco,
>
>Salvador
>
>
>On Tue, 11 Jun 2002, Adherbal Rocha Filho wrote:
>
>> 
>> ajuda:
>> 
>> Mostrar q se o primo p é tal q p==3(mod4), então a equação p^2= a^2 +b^2
>
>> possui solução inteira
>> 
>> mostre q todo quadrado perfeito pode ser representado como soma dos 
>> quadrados de racionais ,naum inteiros, r e s.
>> 
>> valeu!
>> 
>> _
>> Chegou o novo MSN Explorer. Instale já. É gratuito: 
>> http://explorer.msn.com.br
>> 
>> =
>> Instruções para entrar na lista, sair da lista e usar a lista em
>> http://www.mat.puc-rio.br/~nicolau/olimp/obm-l.html
>> O administrador desta lista é <[EMAIL PROTECTED]>
>> =
>> 
>
>=
>Instruções para entrar na lista, sair da lista e usar a lista em
>http://www.mat.puc-rio.br/~nicolau/olimp/obm-l.html
>O administrador desta lista é <[EMAIL PROTECTED]>
>=
>

[]'s, Yuri
ICQ: 64992515


--
Use o melhor sistema de busca da Internet
Radar UOL - http://www.radaruol.com.br



=
Instruções para entrar na lista, sair da lista e usar a lista em
http://www.mat.puc-rio.br/~nicolau/olimp/obm-l.html
O administrador desta lista é <[EMAIL PROTECTED]>
=



[obm-l] Re: [obm-l] t. dos nºs

2002-06-11 Thread yurigomes

 Obs: o teorema anterior afirma que existem INTEIROS a e b.
 No problema p^2= a^2 + b^2 tem (0, p) como soluções inteiras. Se formos
procurar soluções naturais, deveremos ter
 p|a^2 + b^2 . Suponha que p não divide a. Então seja c o inverso de a mod.
p ( que existe, pois (a, p) ). Daí, 
 p|(ac)^2 + (bc)^2, donde (bc)^2 = -1 ( mod p ). Mas o símbolo de Legendre
(-1/p) é igual a (-1)^[(p-1)/2], que é -1 ( pois   (p-1)/2 é ímpar ), absurdo!!
 Logo, p|a e p|b e assim, se a e b são maiores que zero, temos a^2 + b^2
> p^2.

-- Mensagem original --

>
>
>O primeiro problema so pode ter solucao se p=4n+1. 
>
>Para ver isso, observe que a deve ser par e b impar. Logo a^2+b^2 e da
>forma: 4c^2+4d^2+4c+1, que e da forma 4n+1.
>
>De fato todo primo da forma 4n+1 se escreve de um unico jeito como a soma
>de 2 quadrados. Tem um livro chamado "100 great elementary problems: Their
>history and solutions" Heinrich Dorrie, que tem essa prova e muitas outras
>bacanas. Alias esse livro apresenta as "melhores" provas de cada
>problema. E da Dover e nao e dificil de achar.
>
>
>Abraco,
>
>Salvador
>
>
>On Tue, 11 Jun 2002, Adherbal Rocha Filho wrote:
>
>> 
>> ajuda:
>> 
>> Mostrar q se o primo p é tal q p==3(mod4), então a equação p^2= a^2 +b^2
>
>> possui solução inteira
>> 
>> mostre q todo quadrado perfeito pode ser representado como soma dos 
>> quadrados de racionais ,naum inteiros, r e s.
>> 
>> valeu!
>> 
>> _
>> Chegou o novo MSN Explorer. Instale já. É gratuito: 
>> http://explorer.msn.com.br
>> 
>> =
>> Instruções para entrar na lista, sair da lista e usar a lista em
>> http://www.mat.puc-rio.br/~nicolau/olimp/obm-l.html
>> O administrador desta lista é <[EMAIL PROTECTED]>
>> =
>> 
>
>=
>Instruções para entrar na lista, sair da lista e usar a lista em
>http://www.mat.puc-rio.br/~nicolau/olimp/obm-l.html
>O administrador desta lista é <[EMAIL PROTECTED]>
>=
>

[]'s, Yuri
ICQ: 64992515


--
Use o melhor sistema de busca da Internet
Radar UOL - http://www.radaruol.com.br



=
Instruções para entrar na lista, sair da lista e usar a lista em
http://www.mat.puc-rio.br/~nicolau/olimp/obm-l.html
O administrador desta lista é <[EMAIL PROTECTED]>
=



[obm-l] Prova da IMC - 1o. dia

2004-07-25 Thread yurigomes

 Oi pessoal,
 
  O Thiago Barros teve a paciencia de redigir os enunciados dos problemas
do 1o. dia da IMC - 2004 para que vcs pudessem ver e pediu que eu encaminhasse-as
para a lista. Os enunciados estao abaixo.

  Ate mais,
 Yuri

>1) Let S be an infinite set of real numbers such that
>|s_1 + s_2 + ... + s_k| for every finite subset
>{s_1,s_2,...,s_k} of S. Show that S is countable.
>
>2)Let P(x) = x^2 - 1. How many distinct real solutions
>does the following equation have:
>P(P(...(P(x))...)) = 0? [com P sendo aplicado 2004
>vezes]
>
>3) Let S_n be the set of all sum x_1+x_2+...x_n, where
>n>=2, 0<=x_1,...,x_n<="pi"/2 and
>sin(x_1) + sin(x_2) + ... + sin(x_n) = 1
>a) Show that S_n is an interval.
>b)Let l_n be the length of S_n. Find lim(n->infinito)(l_n). 
>
>4)Suppose n>=4 and let M be a finite set of n points in
>R^3, no four of which lie in a plane. Assume that the
>points can be coloured black or white so that any of
>the sphere which intersect M in at least four points have
>the property that exactly half of the points in the
>intersection  of M and the sphere are white. Prove that
>all of the points in M lie on one sphere.
>
>5) Let X be a set of  binomial(2k-4, k-2) + 1  real numbers,
>k>=2. Prove that there exists a monotone sequence x_1, x_2, ..., x_k in
X such that |x_{i+1} - x_1| >= 2|x_i - x_1|
>for all i = 2,...,k-1.
>
>6) For every complex number  z != 0,1 define
>   f(z) := sum((log z)^(-4)),
>where the sum is over all branches  of the complex
>logarithm.
>a) Show that there are two polynomials P and Q such
>that f(z) = P(z)/Q(z) for all z in C\{0,1}
>b) Show that for all z in C\{0,1}
>f(z)=z(z^2 + 4z + 1)/6(z-1)^4.
 
__
Acabe
>com aquelas janelinhas que pulam na sua tela.
AntiPop-up UOL - É grátis!
http://antipopup.uol.com.br/

>
>

Até mais, 

Yuri



--
Use o melhor sistema de busca da Internet
Radar UOL - http://www.radaruol.com.br




=
Instruções para entrar na lista, sair da lista e usar a lista em
http://www.mat.puc-rio.br/~nicolau/olimp/obm-l.html
=


[obm-l] Re: [obm-l] Prova da IMC - 1o. dia (Correcao)

2004-07-25 Thread yurigomes
 Tem soh um errinho pequeno porem decisivo no enunciado do prob 1. Embaixo
estah ajeitado.

-- Mensagem original --

>
> Oi pessoal,
> 
>  O Thiago Barros teve a paciencia de redigir os enunciados dos problemas
>do 1o. dia da IMC - 2004 para que vcs pudessem ver e pediu que eu encaminhasse-as
>para a lista. Os enunciados estao abaixo.
>
>  Ate mais,
> Yuri
>
>>1) Let S be an infinite set of real numbers such that
>>|s_1 + s_2 + ... + s_k| < 1 for every finite subset
>>{s_1,s_2,...,s_k} of S. Show that S is countable.
>>
>>2)Let P(x) = x^2 - 1. How many distinct real solutions
>>does the following equation have:
>>P(P(...(P(x))...)) = 0? [com P sendo aplicado 2004
>>vezes]
>>
>>3) Let S_n be the set of all sum x_1+x_2+...x_n, where
>>n>=2, 0<=x_1,...,x_n<="pi"/2 and
>>sin(x_1) + sin(x_2) + ... + sin(x_n) = 1
>>a) Show that S_n is an interval.
>>b)Let l_n be the length of S_n. Find lim(n->infinito)(l_n). 
>>
>>4)Suppose n>=4 and let M be a finite set of n points in
>>R^3, no four of which lie in a plane. Assume that the
>>points can be coloured black or white so that any of
>>the sphere which intersect M in at least four points have
>>the property that exactly half of the points in the
>>intersection  of M and the sphere are white. Prove that
>>all of the points in M lie on one sphere.
>>
>>5) Let X be a set of  binomial(2k-4, k-2) + 1  real numbers,
>>k>=2. Prove that there exists a monotone sequence x_1, x_2, ..., x_k in
>X such that |x_{i+1} - x_1| >= 2|x_i - x_1|
>>for all i = 2,...,k-1.
>>
>>6) For every complex number  z != 0,1 define
>>   f(z) := sum((log z)^(-4)),
>>where the sum is over all branches  of the complex
>>logarithm.
>>a) Show that there are two polynomials P and Q such
>>that f(z) = P(z)/Q(z) for all z in C\{0,1}
>>b) Show that for all z in C\{0,1}
>>f(z)=z(z^2 + 4z + 1)/6(z-1)^4.
> 
>__
>Acabe
>>com aquelas janelinhas que pulam na sua tela.
>AntiPop-up UOL - É grátis!
>http://antipopup.uol.com.br/
>
>>
>>
>
>Até mais, 
>
>Yuri
>
>
>
>--
>Use o melhor sistema de busca da Internet
>Radar UOL - http://www.radaruol.com.br
>
>
>
>
>=
>Instruções para entrar na lista, sair da lista e usar a lista em
>http://www.mat.puc-rio.br/~nicolau/olimp/obm-l.html
>=
>

Até mais, 

Yuri



--
Use o melhor sistema de busca da Internet
Radar UOL - http://www.radaruol.com.br




=
Instruções para entrar na lista, sair da lista e usar a lista em
http://www.mat.puc-rio.br/~nicolau/olimp/obm-l.html
=


[obm-l] Prova da IMC - 1o. dia (correcao)

2004-07-25 Thread yurigomes
 Bem, num sei se a correcao que fiz no ultimo email vai chegar, entaum estou
novamente corrigindo o enunciado do problema 1.

 
>1) Let S be an infinite set of real numbers such that
>|s_1 + s_2 + ... + s_k| < 1 for every finite subset
>{s_1,s_2,...,s_k} of S. Show that S is countable.
>
>2)Let P(x) = x^2 - 1. How many distinct real solutions
>does the following equation have:
>P(P(...(P(x))...)) = 0? [com P sendo aplicado 2004
>vezes]
>
>3) Let S_n be the set of all sum x_1+x_2+...x_n, where
>n>=2, 0<=x_1,...,x_n<="pi"/2 and
>sin(x_1) + sin(x_2) + ... + sin(x_n) = 1
>a) Show that S_n is an interval.
>b)Let l_n be the length of S_n. Find lim(n->infinito)(l_n). 
>
>4)Suppose n>=4 and let M be a finite set of n points in
>R^3, no four of which lie in a plane. Assume that the
>points can be coloured black or white so that any of
>the sphere which intersect M in at least four points have
>the property that exactly half of the points in the
>intersection of M and the sphere are white. Prove that
>all of the points in M lie on one sphere.
>
>5) Let X be a set of binomial(2k-4, k-2) + 1 real numbers,
>k>=2. Prove that there exists a monotone sequence x_1, x_2, ..., x_k in
X such that |x_{i+1} - x_1| >= 2|x_i - x_1|
>for all i = 2,...,k-1.
>
>6) For every complex number z != 0,1 define
> f(z) := sum((log z)^(-4)),
>where the sum is over all branches of the complex
>logarithm.
>a) Show that there are two polynomials P and Q such
>that f(z) = P(z)/Q(z) for all z in C\{0,1}
>b) Show that for all z in C\{0,1}
>f(z)=z(z^2 + 4z + 1)/6(z-1)^4. 


Até mais, 

Yuri



--
Use o melhor sistema de busca da Internet
Radar UOL - http://www.radaruol.com.br




=
Instruções para entrar na lista, sair da lista e usar a lista em
http://www.mat.puc-rio.br/~nicolau/olimp/obm-l.html
=


[obm-l] Solucoes da IMC

2004-07-27 Thread yurigomes
 Oi Marcio e pessoal da lista,
  
 Estamos aguardando a recorrecao que serah amanha, visto que acho que terah
muita mudanca depois disso. Qto aas suas solucoes:

PRIMEIRO DIA 

Prob 1: todo mundo fez considerando subconjuntos contidos nos intervalos
[1/(k+1), 1/k] e o reciproco negativo. Cada subconjunto desses sera finito,
de modo que a soma enumeravel de conjuntos finitos eh enumeravel.

 Prob 2: O Alex e mais alguns fizeram desse jeito. Eu fiz uma inducao um
pouco mais complicada e indo de um em um.

 Prob 3: basicamente igual.

 Prob 4: tente considerar tres pontos de uma mesma cor. Fazendo isso, a
quantidade de pontos da cor oposta sera maior do que a da cor dos tres pontos.
Entaum se vc tomar outros tres pontos da cor oposta, voce chegara a um absurdo.

 Prob 5: existe uma generalizacao, mas nao tou lembrado agora. A ideia eh
dividir o intervalo no meio e usar inducao.

 Prob 6: ainda naum sei, mas acho que seja do jeito que vc fez.

SEGUNDO DIA

 Prob 1: considere submatrizes 2x2 de cada uma das duas matrizes e veja
o que acontece.

 Prob 2: Considere F = int(sqrt(f)) e G = int(sqrt(g)), mostre que F e G
saum convexas, tem pontos finais iguais e F <= G. Analise o comprimento
de arco do grafico... Praticamente feita!

 Solucao do Alex: 
 LEMA: Seja f continua tal que int(a, b)(f) = 0, int(x, b)(f) >= 0 e seja
h de classe C^1 nao decrescente. Entaum int(a, b)(f*h) >= 0. 

  PV: Temos que int(a, b)[int(x, b)(f(t)*h'(x))dt]dx >= 0, pois h'(x) >=
0. Troque a ordem de integracao para concluir o lema. 

  SOLUCAO DO PROBLEMA: use 
   f := sqrt(f) - sqrt(g) e 
   h := (sqrt(f) + sqrt(g))(sqrt(1 + f) + sqrt(1 + g)).
   h nao eh C^1 mas eh nao decrescente e portanto pode ser aproximada por
funcoes C^1 nao decrescentes (Stone-Weiertrass). Fim.

 Prob 3: existem varias solucoes. Eu usei um polinomio e trabalhei com as
raizes n-esimas da unidade rotacionadas de um fator pras contas darem certo.
A maioria integrou a funcao distancia a cada p_i sobre a circunferencia
e o Humberto considerou os vertices de um quadrado e mostrou que a soma
das distancias de cada p_i a esses vertices eh sempre >= 4 (na verdade,
soh sao necessarios dois pontos diametralmente opostos).

 Prob 4: apos pensar um pouco, suspeita-se que os autovalores sao lamba_i
+ lambda_j com multiplicidade m_i*m_j.

 Prob 5: soh eu fiz do Brasil, usando que |lnx| >= 1 - x para todo x em
[0, 1]. Com isso, vc fica com uma integral que dah pra calcular e que dah
exatamente 1.

 Prob 6: a solucao oficial eh um argumento combinatorio escroto, como tah
me dizendo o Alex aqui do lado. Ninguem ainda tentou entender, hehe...

  As pontuacoes parciais jah sairam. Por enquanto quem estah em primeiro
lugar fez 190 ptos. No primeiro dia, o que fez mais foi 110. Isso mostra
a maior dificuldade do segundo dia, que nao tava com nenhuma questao muito
facil. A segunda questao estah muito mal colocada (acho que do Brasil soh
o Humberto fez). Achei a tres a mais facil do segundo dia. A solucao oficial
eh a que o Claudio fez. Ok, acho que eh soh isso. Amanha esperamos mandar
um email com boas noticias, hehe... 
 
Yuri e Alex
 
-- Mensagem original --

>   Oi gente.. Tentei fazer as questoes do 1o dia da imc. Estamos ansiosos
>por noticias de como o pessoal esta indo na prova! Ao contrario da IMO,
na
>IMC nao eh o lider que corrige as provas do pais. Ele participa da banca
>de
>uma determinada questao e depois participa da revisao de notas dos seus
>alunos. Por esse motivo, seria interessante que voces postassem as
>principais solucoes para que o pessoal da lista possa ajudar..
>Quem nao quiser ver as solucoes, pare de ler por aqui. Vou escrever
>minhas solucoes, leiam com atencao e me avisem se notarem algo
>errado. O 1,2,3 eram mais faceis. O 5 eu escrevi 2 linhas sem mto nexo
la
>em
>baixo :) e no 6 eu usei um resultado nao obvio de analise complexa (e pulei
>algumas contas que teriam de ser feitas na prova).
>Ja o 4 eu demorei bastante (um pouco mais q o 2o tempo inteiro do jogo
BrasilxArgentina)
>e achei que tinha feito. Mas depois eu fui ver a solucao do mathlinks e
vi
>que era bem curta, mto simples, e comeco a achar que a minha esta errada
>(alem de ser completamente gigante). 
>Em tempo: eu acabei de ver a prova do 2o dia. A primeira eh bem simples,
>usa uma ideia analoga a que foi usada na 1 do 2o dia do ano passado :)
Mas
>as outras parecem estar bem mais dificeis do que as de hoje!! Nao tenho
ideia
>alguma para a 2 por exemplo.. 
>Abracos,
>Marcio
>
>
>> >1) Let S be an infinite set of real numbers such that
>> >|s_1 + s_2 + ... + s_k| for every finite subset
>> >{s_1,s_2,...,s_k} of S. Show that S is countable
>
>Escreva S = A U B, onde A e B sao respectivamente os subconjuntos de
>nao-negativos e nao-positivos de S.
>Vamos provar que A e B sao enumeraveis e portanto S tmb é (por ser
uniao
>de 2 enumeraveis).
>A enumeracao eh a seguinte (A e B sao analogos). Se A eh finito, eh
>simples. Caso contrario,coloque x_1 = max{A}, x_2 = max{A

[obm-l] =?iso-8859-1?Q?Re=3A=20=5Bobm=2Dl=5D=20questao=20do=20ITA=20furada?=

2004-12-16 Thread yurigomes
 Oi pessoal,
  
 Pelo visto a prova do ITA tá dando muita controvérsia. A solução da questão
abaixo (questão 30) está com um pequeno erro, uma vez que a=1 também gera
3 soluções, que são 0, 1 e 2. 
 Divulgamos a prova com soluções em www.c7s.com.br. Acho que muitas dessas
controvérsias foram solucionadas. Dêem uam conferida. 

 Um abraço, 
 
 Yuri 

-- Mensagem original --

>> aa, entao deve ser por isso que o anglo ainda nao divulgou a resolucão
>da questao 30... eles devem estar tentando considerar que x pode ser complexo...
>>
>> Questão 30. Determine todos os valores reais de a para os quais a equação
>>
>> (x-1)^2 = |x - a|
>
>
>x^2-2x+1=|x-a|
>
>Se x>=a: x^2-3x+(1+a)=0(*)
>
>Se x
>I) delta(*)=0 e delta(**)=!0=>9-4(1+a)=0=>a=4/5 e -3+4a=!0 (V)
>
>II) delta(**)=! e delta(*)=0=>-3+4a=0=>a=3/4 e 9-4(1+3/4)=!0(V)
>
>Assim os valores de a são 4/5 e 3/4
>
>
>x=[3+/-sqrt(5-4a)]/2(*)
>
>x=[1+/-sqrt(4a-3)]/2(**)
>
>Se o valor de x de um membro for complexo o do otro terá que ser tambem,
>logo nunca haverá um número ímpar de soluções.
>
>Soh vou prestar ITA ano que vem, a prova de Química como sempre mta Físico-Química.
>
>
>
>
>>
>> admita exatamente três soluções distintas.
>>
>> hehehe, eles devem estar tendo moh trabalhão...
>>
>>
>> On Wed, Dec 15, 2004 at 07:47:42PM -0200, Claudio Buffara wrote:
>> > on 15.12.04 19:21, Fabio Niski at [EMAIL PROTECTED] wrote:
>> >
>> > > A questao 11 do ITA "No desenvolvimento de (ax^2 + -2bx + c + 1)^5
>
>> > > obtem-se um polinomio p(x) cujos coeficientes somam 32. Se 0 e -1
sao
>> > > raizes de p(x), entao a soma a + b + c é igual a
>> > > a) -1/2 b) -1/4 c) 1/2 d)1 e)3/2
>> > >
>> > > Pelo o que eu vi, Etapa, Poliedro e Objetivo marcaram A.
>> > > O Anglo observou corretamente que existem 5 possiveis valores possiveis
>> > > pra soma e a questao deveria ser cancelada.
>> > >
>> > Essa eh complicada. Nao ha nada no enunciado que diga que a deve ser
>real,
>> > apesar dessa ser uma hipotese razoavel.
>> >
>> > Qual foi o veredito?
>> >
>> >
>> >
>> >
>> >
>> > =
>> > Instruções para entrar na lista, sair da lista e usar a lista em 
>> > http://www.mat.puc-rio.br/~nicolau/olimp/obm-l.html
>> > =
>> =
>
>> Instruções para entrar na lista, sair da lista e usar a lista em
>> http://www.mat.puc-rio.br/~nicolau/olimp/obm-l.html
>> =
>
>>
>Atenciosamente,
>Osvaldo Mello Sponquiado
>Engenharia Elétrica, 2ºano
>UNESP - Ilha Solteira 
 
__
Acabe
>com aquelas janelinhas que pulam na sua tela.
AntiPop-up UOL - É grátis!
http://antipopup.uol.com.br/

>

Até mais, 

Yuri



--
Use o melhor sistema de busca da Internet
Radar UOL - http://www.radaruol.com.br




=
Instruções para entrar na lista, sair da lista e usar a lista em
http://www.mat.puc-rio.br/~nicolau/olimp/obm-l.html
=


[obm-l] Re: [obm-l] Variáveis complexas

2005-01-02 Thread yurigomes
 Oi Tertuliano,

1) Suponha que f(z) =! 0, para todo z em U. Considere g = 1/f. Então g tem
um máximo local, a dizer z = a, e portanto deve ser constante.

2) Vamos mostrar que f^(n+1)(z) = 0, para todo z em U. De fato, tome r >
max{R, |z|}. Então pela fórmula integral de Cauchy temos:

f^(n+1)(z) = [(n+1)!/2.pi.i]int(|w|=r)(f(w)dw/(w-z)^(n+2)) =>
|f^(n+1)(z)| <= [(n+1)!/2.pi]int(|w|=r)(|f(w)||dw|/|w-z|^(n+2)) 
<= [(n+1)!/2.pi](M.r^n).(2.pi.r)/(r-|z|)^(n+2)= 
= M.(n+1)!.r^(n+1)/(r-|z|)^(n+2) =
= M.(n+1)!/[r(1-|z|/r)^(n+2)] -> 0 qdo r -> infty.
 Isso garante que f^(n+1) é ltda e portanto, pelo Teorema de Liouville,
é constante. Assim, f é um polinômio de grau <= n.  

 Abraços,
 
Yuri
-- Mensagem original --

>Feliz ano novo para todos da lista. Gostaria que me ajudassem nesses problemas:
> 
>1) Seja f : U em C (complexos) uma funcao holomorfa, onde u é um domínio.
>Suponha q exista um ponto a em U tq /f(a)/ é menor ou igual a /f(z)/ para
>todo z em U. Mostre q ou f(a) = 0 ou f é uma funcao constante. 
>Obs.: /x/ representa a norma de x.
> 
>2) Seja f uma funcao inteira (holomorfa em todo plano complexo C) e suponha
>q existem M, R positivos e n maior ou igual a 1 tq /f(z)/ é menor ou igual
>a M/z/^n para /z/ maior ou igual a R. Mostre q f é um polinomio cujo grau
>máximo é n.
> 
> 
>Grato,
>Tertuliano Carneiro   
>
>   
>-
>Yahoo! Acesso Grátis - Internet rápida e grátis. Instale o discador do
Yahoo!
>agora.


Até mais, 

Yuri



--
Use o melhor sistema de busca da Internet
Radar UOL - http://www.radaruol.com.br




=
Instruções para entrar na lista, sair da lista e usar a lista em
http://www.mat.puc-rio.br/~nicolau/olimp/obm-l.html
=


[obm-l] Re: [obm-l] [OFF] Sir Michael Atiyah no Brasil

2005-03-17 Thread yurigomes
 Só corrigindo, ele foi Medalha Fields em 66, até porque ele já tem mais
de 70 anos!!
 
-- Mensagem original --

>Para quem se interessa por matemática...Sir Michael Atiyah, um dos
>maiores matemáticos da atualidade, detentor da melhada Fields de 96 e
>do prêmio Abel de 04, fará uma palestra gratuita no Rio de Janeiro (na
>UFRJ da praia vermelha), dia 17/03 as 17:30hs, com o tema "The Nature
>of Space"..
>
>http://olharvirtual.ufrj.br/05_03_10_cursos.htm
>
>[]s
>daniel
>
>-- 
>"A essência da Matemática reside na sua liberdade." (G. Cantor)
>
>=
>Instruções para entrar na lista, sair da lista e usar a lista em
>http://www.mat.puc-rio.br/~nicolau/olimp/obm-l.html
>=
>

Até mais, 

Yuri



--
Use o melhor sistema de busca da Internet
Radar UOL - http://www.radaruol.com.br




=
Instruções para entrar na lista, sair da lista e usar a lista em
http://www.mat.puc-rio.br/~nicolau/olimp/obm-l.html
=


[obm-l] Cone Sul 2005 - prova e resultados

2005-05-28 Thread yurigomes

Oi pessoal,

Notícias e resultados da Cone Sul... Os problemas estão no final do e-mail.


1o e-mail:

Ai vai o primeiro dia da Cone Sul aqui na Bolívia. Foi um dia fácil, e espero
que os garotos tenham ido bem, ainda não vi as provas deles. O problema
1 é da Bolívia, o 2 é nosso (Cícero), e o 3 é da Argentina.

Um abraço, 
Emanuel.

2o e-mail:

Oi galera, aí vai o segundo dia da Cone Sul. O problema 4 é nosso (acho
que do Yuri...), o 5 é da Argentina e o 6 é do Peru (versão original r =
1/14, que melhoramos aqui para r=1/20, para que caísse pelo menos um problema
um pouco mais difícil na prova...)

Nosso problema de ontem foi bom pra nós, essencialmente todos fizeram completos
(talvez percamos uns pontinhos com o Rafael...). O 1 tbm todos fizeram e
o 3 só o Guilherme fez completo (Edson e Henrique podem ganhar 3 ou 4 pontos).
Se comenta que 4 pessoas fecharam o dia 1 (1 Brasil, 1 Argentina e 2 Peru).

Agora os garotos acabaram de sair do segundo dia e segundo eles todos fizeram
o 4 e o 5 (fáceis) menos o Edson que jurou pra mim que não tinha conseguido
fazer o 5, mas eu não acredito, só vou acreditar quando receber sua prova
mais tarde... No 6 eles não fizeram e me disseram que escreveram pouco,
acho que vai ser difícil ganhar pontos e talvez não saia ouro pra nós, já
que as más línguas dizem que um peruano vai fechar (o cara já foi prata
na Ibero...), mas veremos amanha. Tudo caminha para que os brasileiros todos
sejam medalhados...

Agora a comédia, hoje ocorreu um golpe de Estado na Bolívia, La Paz esta
sitiada pelo exército e todos os vôos domésticos estão cancelados por hora...
talvez tenhamos que ficar um pouquinho mais por aqui (temos um trecho domestico
na volta Sucre - Santa Cruz). Vou averiguar melhores esses acontecimentos...

Um abraço. Emanuel.

3o e-mail:

Ola Galera,
Ontem foi um dia de muito trabalho pra mim e pro Davi, começamos a coordenação
às 8 da manhã e terminamos só às 2 da tarde, e pela primeira vez vimos (ou
melhor levamos) um problema à apreciação do tribunal à noite (o problemas
do Guilherme). O pessoal da banca era extremamente jeca, se enrolavam todos,
melhor pra nós, que ao final conseguimos tudo que queríamos e um pouco mais.


No final o resultado me pareceu excelente tendo em vista a jovem equipe
que levamos: 2 ouros e 2 pratas (e olha que a galera não aloprou em distribuir
medalhas não: foram 3 ouros e 5 pratas como manda o figurino e seriam só
8 bronzes se depois não viessem 2 bolivianos e 1 uruguaio empatados, então
foram 11 bronzes).

Nossas notas foram (só por curiosidade, em parênteses vão as notas, que
foram diferentes, inicialmente propostas pelas bancas):
BRA1-Edson -10-10-3(1)-10-2(0)-1-Total  36 (33) (prata) 
BRA2-Guilherme -10-10(0)-10(2)-10(0)-10- 2-  Total  52 (24) (ouro) 
BRA3-Henrique-10-10(7)-4(2)-10-10- 8 - - Total  52 (41) (ouro) 
BRA4-Rafael-10-9(4)-2(0)-10-10(4)-2(0)?Total 43(28) (prata)
Em resumo o resultado final foi assim: 
Ouro 
Per2 - 53 
Bra2 - 52 
Bra 3 - 52
Prata 
Arg2 - 49 
Bra4 - 43 
Per 1- 40 
Bra 1- 36 
Chi 1- 35
Bronze 
Arg1 - 34 
Par1 -33 
Chi4- 31 
Per - 31 
Per - 30 
Arg - 30 
Arg - 30 
Uru- 28 
Bol - 25 
Bol - 25 
Uru - 25

Observações interessantes: a maior nota do prob6 foi do Henrique, 8, mas
acho que ele ainda não sabe fazer o problema... e a segunda maior foi um
3. 
O Edson não tinha feito mesmo o prob5... ganhamos 2 pontos por algo que
o bocó escreveu e riscou por cima... 
O prob2 do Guilherme foi para o tribunal pois a banca não aceitou sua solução
por Geometria Analítica e Trigonometria (¿donde estan Euclides e la geometría
plana?), queriam dar zero.
Importante: Optamos aqui por não falar aos garotos o resultado, achamos
que eles vão ficar surpresos na cerimônia de premiação que será às 7 horas
aqui, 8 do Brasil, por isso não divulguem os resultados a ninguém relacionado
a eles (tipo família, diretor de escola...) pra que eles não liguem pra
casa e saibam.
Um abraço. Emanuel e Davi.


PROBLEMA 1
Considere a seguinte seqüência:
a_1 = último dígito da soma dos dígitos do número 2005
a_2 = último dígito da soma dos dígitos do número 20052005
a_3 = último dígito da soma dos dígitos do número 200520052005
...
Calcule: a_1+a_2+a_3+···+a_2005

PROBLEMA 2
Seja ABC um triângulo acutângulo e sejam AN, BM e CP as alturas relativas
aos lados BC, CA e AB, respectivamente. Sejam R, S as projeções de N sobre
os lados AB, CA, respectivamente, e Q, W as projeções de N sobre as alturas
BM e CP, respectivamente. 
(a)Mostre que R, Q, W, S são colineares;
(b)Mostre que MP = RS ? QW.

PROBLEMA 3
A unidade monetária de um certo país se chama reo, e todas as moedas que
circulam são de números inteiros de reos. Em um grupo de três pessoas, cada
uma tem 60 reos em moedas (mas não se sabe que tipo de moedas cada uma tem).
Cada uma das três pessoas pode pagar a cada uma das outras qualquer valor
inteiro entre 1 e 15 reos, inclusive, talvez com troco. Mostre que as três
pessoas em conjunto podem pagar exatamente (sem troco) qualquer valor inteiro
entre 45

[obm-l] Primeiro dia - XX Ibero

2005-09-28 Thread yurigomes
Oi pessoal,

Estou trazendo notícias da Ibero. 

Nossa viagem foi um pouco complicada. Qdo chegamos no aeroporto de Guarulhos
descobrimos que nosso voo, que partiu de RJ, teria uma troca para uma aeronave
menor, e isso causou um overbooking enorme. 

Entao todos que partiam de SP, eu, Gabriel, Thiago e Rafael, tivemos que
arranjar outros voos. O Thomaz acabou indo no voo certo, pq saiu de RJ.

Nós fomos para Santiago, dormimos lá, e no outro dia seguimos viagem para
Bogotá, e por fim Cartagena. Pelo menos íamos chegar um dia antes, entao
todos puderam descansar bem.

Sobre a prova: o 1o dia foi ontem, e o 2o está acontecendo nesse instante.
Segue abaixo a prova de ontem:

PROBLEMA 1
Determine todas as ternas de números reais (x,y,z) tais que
xyz = 8  ,
x^2y + y^2z + z^2x = 73  ,
x(y-z)^2 + y(z-x)^2 + z(x-y)^2 = 98.

PROBLEMA 2
Uma pulga salta sobre pontos inteiros da reta numérica. Em seu primeiro
movimento ela salta do ponto 0 ao ponto 1. Em seguida, se em um movimento
ela salta do ponto A e cai no ponto B, entao no proximo movimento ela salta
do ponto B e cai em um dos pontos
B + (B-A) - 1  ,  B + (B-A)  ,  B + (B-A) + 1 .

Mostre que se a pulga caiu duas vezes sobre o ponto n, n inteiro positivo,
entao ela fez pelo menos t movimentos, onde t é o menor inteiro maior ou
igual a 2.n^(1/2).

PROBLEMA 3
Seja p>3 um primo. Se
\sum_{i=1}^{p-1} 1/(i^p) = n/m,

onde mdc(n,m)=1, mostre que p^3 divide n.

Como foram os meninos: parece que todos fizeram o 1 e 3 bem rápido, e ficaram
o resto da prova no 2. O Gabriel e Rafael conseguiram fazer, o Thomaz acho
que chegou relativamente perto e o Thiago nao fez muita coisa. 

Hj comecaremos a correccao. Qto já tivermos algo certo enviaremos pra vcs.

Abraccos,

Yuri



Até mais, 

Yuri




=
Instruções para entrar na lista, sair da lista e usar a lista em
http://www.mat.puc-rio.br/~nicolau/olimp/obm-l.html
=


[obm-l] Segundo dia - XX Ibero

2005-09-28 Thread yurigomes
Estou enviando agora os problemas do segundo dia. Ainda nao sei como os
meninos foram pq desde depois do almocco eles estao na prova por equipes
e nem chegaram a voltar pro hotel.

O problema 5 é do Brasil (meu e do Davi). =)) 

PROBLEMA 4
Dados dois inteiros positivos a e b, denota-se por (a mod b) o resto da
divisao de a por b, que é um dos números 0,1, ..., b-1. Determine todos
os pares de números (a,p) tais que p é primo e 
 
(a mod p) + (a mod 2p) + (a mod 3p) + (a mod 4p) = a + p.
 
PROBLEMA 5
Seja O o circuncentro de um triangulo acutangulo ABC e A_1 um ponto do arco
menor BC da circunferencia circunscrita ao triangulo ABC. Sejam A_2 e A_3
pontos dos lados AB e AC, respectivamente, tais que vale a igualdade de
angulos
BA_1A_2 = OACeCA_1A_3 = OAB. 
Demonstre que a reta A_2A_3 passa pelo ortocentro do triangulo ABC.
 
PROBLEMA 6
Dado um inteiro positivo n, num plano consideram-se 2n pontos alinhados
A_1, A_2, ..., A_2n. Pinta-se cada ponto de azul ou vermelho de acordo com
o seguinte procedimento:
no plano dado sao traccadas n circunferencias disjuntas duas a duas, com
diametros de extremos A_i e A_ j. Cada A_k, 1 <= k <= 2n, pertence exatamente
a uma circunferencia. Os pontos sao pintados de modo que dois pontos de
uma mesma circunferencia ficam com a mesma cor.
Determine o numero de coloraccoes distintas dos 2n pontos que se podem obter
ao variar as n circunferencias e a distribuiccao das duas cores.
 
Abraccos,
 
Yuri
 


-- Mensagem original --

>Oi pessoal,
>
>Estou trazendo notícias da Ibero. 
>
>Nossa viagem foi um pouco complicada. Qdo chegamos no aeroporto de Guarulhos
>descobrimos que nosso voo, que partiu de RJ, teria uma troca para uma aeronave
>menor, e isso causou um overbooking enorme. 
>
>Entao todos que partiam de SP, eu, Gabriel, Thiago e Rafael, tivemos que
>arranjar outros voos. O Thomaz acabou indo no voo certo, pq saiu de RJ.
>
>Nós fomos para Santiago, dormimos lá, e no outro dia seguimos viagem para
>Bogotá, e por fim Cartagena. Pelo menos íamos chegar um dia antes, entao
>todos puderam descansar bem.
>
>Sobre a prova: o 1o dia foi ontem, e o 2o está acontecendo nesse instante.
>Segue abaixo a prova de ontem:
>
>PROBLEMA 1
>Determine todas as ternas de números reais (x,y,z) tais que
>xyz = 8  ,
>x^2y + y^2z + z^2x = 73  ,
>x(y-z)^2 + y(z-x)^2 + z(x-y)^2 = 98.
>
>PROBLEMA 2
>Uma pulga salta sobre pontos inteiros da reta numérica. Em seu primeiro
>movimento ela salta do ponto 0 ao ponto 1. Em seguida, se em um movimento
>ela salta do ponto A e cai no ponto B, entao no proximo movimento ela salta
>do ponto B e cai em um dos pontos
>B + (B-A) - 1  ,  B + (B-A)  ,  B + (B-A) + 1 .
>
>Mostre que se a pulga caiu duas vezes sobre o ponto n, n inteiro positivo,
>entao ela fez pelo menos t movimentos, onde t é o menor inteiro maior ou
>igual a 2.n^(1/2).
>
>PROBLEMA 3
>Seja p>3 um primo. Se
>\sum_{i=1}^{p-1} 1/(i^p) = n/m,
>
>onde mdc(n,m)=1, mostre que p^3 divide n.
>
>Como foram os meninos: parece que todos fizeram o 1 e 3 bem rápido, e ficaram
>o resto da prova no 2. O Gabriel e Rafael conseguiram fazer, o Thomaz acho
>que chegou relativamente perto e o Thiago nao fez muita coisa. 
>
>Hj comecaremos a correccao. Qto já tivermos algo certo enviaremos pra vcs.
>
>Abraccos,
>
>Yuri
>
>
>
>Até mais, 
>
>Yuri
>
>
>
>
>=
>Instruções para entrar na lista, sair da lista e usar a lista em
>http://www.mat.puc-rio.br/~nicolau/olimp/obm-l.html
>=
>

Até mais, 

Yuri




=
Instruções para entrar na lista, sair da lista e usar a lista em
http://www.mat.puc-rio.br/~nicolau/olimp/obm-l.html
=


[obm-l] Resultado - XX Ibero

2005-09-30 Thread yurigomes
Oi pessoal,

Estou enviando agora as pontuaccoes por problemas e os resultados. Bem,
conseguimos novamente o máximo que podíamos: 4 ouros! Um deles foi "puntaje
perfecto", o Gabriel. As pontuaccoes foram as seguintes:
  
1 2  34 56   TOTAL
BRA 1 (Thiago)  6 0  67 77  33
BRA 2 (Thomaz)  7 6  67 70  33
BRA 3 (Rafael)  7 7  7 7 70 35
BRA 4 (Gabriel) 7 7  7 7 77 42

Os cortes foram:
Ouro - 33
Prata - 22
Bronze - 10

A competiccao teve 22 paises e 85 estudantes. No total foram 8 ouros, 17
pratas, 23 bronzes e 5 menccoes honrosas. 3 pessoas fecharam: Bujokas, um
da Colombia e outro do Mexico. Os outros dois ouros foram da Argentina (35)
e outro do Mexico (33).

Ah... por uns cinco minutos vencemos a Copa Porto Rico. Mostraram a tabela
e estavamos em primeiro. Mas aí percebeu-se que isso nao era possivel pq
com relacao ao ano passado haviamos diminuido em qtde. de pontos enquanto
o Uruguai havia sempre aumentado nos dois ultimos anos. Entao qdo o responsavel
foi mostrar a formula, ele percebeu que havia colocado uma incognita errada.
Acabou-se o que era doce, hehe...

Com relaccao aa correccao, nao tivemos muitos problemas. Apenas dois problemas
do Thiago (3 e 6), um do Rafael (2) e um do Thomaz (2 tbm) tiveram disparidade
de notas. Mas no fim das contas conseguimos o que queriamos.

Bem, acho que é isso. Parabéns aos estudantes.

Abraccos,

Yuri



Até mais, 

Yuri




=
Instruções para entrar na lista, sair da lista e usar a lista em
http://www.mat.puc-rio.br/~nicolau/olimp/obm-l.html
=


[obm-l] Soluções IME 2006

2005-10-26 Thread yurigomes
Oi pessoal,

Quem estiver interessado nas soluções da prova de matemática do IME 2006
dê uma olhadinha em 

www.c7s.com.br

Tá bem legal!

Abraços,

Yuri

Até mais, 

Yuri




=
Instruções para entrar na lista, sair da lista e usar a lista em
http://www.mat.puc-rio.br/~nicolau/olimp/obm-l.html
=


[obm-l] Ibero 2003

2003-09-18 Thread yurigomes

  Já estão disponíveis os dois dias de prova da Ibero. O link eh:

  http://www.campus-oei.org/oim/xviiioim.htm

 Ateh mais,
  Yuri 

[]'s, Yuri
ICQ: 64992515


--
Use o melhor sistema de busca da Internet
Radar UOL - http://www.radaruol.com.br



=
Instruções para entrar na lista, sair da lista e usar a lista em
http://www.mat.puc-rio.br/~nicolau/olimp/obm-l.html
=


[obm-l] Re: [obm-l] Algebra linear : Wronkisano e indicacao de livro

2003-09-20 Thread yurigomes
 Oi Niski, 
 Eu acho que dá pra fazer pelo wronskiano mesmo. Vamos lá:
 Derivando e^(a(j).x) i vezes, ficamos com a(j)^i.e^(a(j).x). Logo:
  W= W(e^(a(1).x),...,e^(a(n).x))= det((a(j)^(i-1).e^(a(j).x)), onde i representa
a linha e j a coluna. Agora observe que todos os elementos da coluna j têm
e^(a(j).x). Logo podemos colocar esse valor para fora do determinante. Fazendo
isso com todas as colunas, ficamos com 
  W= e^(a(1).x)...e^(a(n).x).det(a(j)^(i-1))
 Mas det(a(j)^(i-1)) é o determinante de Vardemont dos números   a(1),...,a(n),que
é igual 
  Prod(1<=i0, para todo k, segue que W é diferente de zero, como queríamos
demonstrar.
 Ateh mais, 
 Yuri

-- Mensagem original --

>Ola pessoal. Inicialmente, agradeco ao incansavel Claudio Bufarra pela

>resolucao lá da equacao da involute da circunferencia.
>Bom estou com o seguinte problema
>Seja A = {e^a[1].x, e^a[2].x, e^a[3].x, ..., e^a[n]x} *(obs lê-se e 
>elevado a a indicie n vezes x)
>onde a[1] != a[2] != ... != a[n] e pertencem a R.
>Prove que A é L.I.
>
>Eu pensei em contruir o Wronskiano e se ele for identicamente nulo então
>
>seria L.I certo?
>Eu tentei utilizar a definicao de determinante com aqueles somatorios e

>trocas de sinais malucos...sem resultado..
>Alguem poderia provar por gentileza?
>Aproveito o ensejo para pedir aos membros da lista referencias sobre 
>algebra linear e equacoes diferenciais (lineares).
>Agradeco antecipadamente.
>
>Niski
>
>=
>Instruções para entrar na lista, sair da lista e usar a lista em
>http://www.mat.puc-rio.br/~nicolau/olimp/obm-l.html
>=
>

[]'s, Yuri
ICQ: 64992515


--
Use o melhor sistema de busca da Internet
Radar UOL - http://www.radaruol.com.br



=
Instruções para entrar na lista, sair da lista e usar a lista em
http://www.mat.puc-rio.br/~nicolau/olimp/obm-l.html
=


[obm-l] Re: [obm-l] obm - U

2003-10-21 Thread yurigomes

 Vc pode fazer essa desigualdade por Cauchy: observe
  (SOMA{(sr(p_i^3))^2}).(SOMA{((sr(p_i))^2} >=
   (SOMA{sr(p_i^3).sr(p_i})^2  
  Mas o segundo fator do lado esquerdo é igual a SOMA(p_i)=1, e o resultado
segue.
  Outra maneira seria observar que 
   SOMA{p_i^3) = SOMA{p_i^3).SOMA{p_i) = SOMA(p_i^4) + SOMA(i != j){p_i^3.p_j).

  Desenvolvendo o lado direito da desigualdade que vc quer mostrar e cancelando
soma(p_i^4), vc vai querer que  
   SOMA(i != j){p_i^3.p_j) >=  2.SOMA(i < j){p_i^2.p_j^2)
  Por média, p_i^3.p_j + p_i.p_j^3 >= 2p_i^2.p_j^2. Aih basta somar em i,j.
  Ateh mais, 
  Yuri 

-- Mensagem original --

>Oi Nicolau!
>
>E quanto ao problema quatro? Eu chamei de 0 < p_i < 1 a probabilidade de
>sair a face i num lançamento, tendo-se SOMA{p_i} = 1. Eu desenvolvi um
pouco
>o problema e mostrei que ele era equivalente a demonstrar a desigualdades
>SOMA{p_i^3} >= SOMA{p_i^2}^2 com igualdade sse todos p_i = 1/6. Não consegui
>demonstrar esta desigualdade. Quando vale este primeiro passo? ;) Como
se
>demonstra esta desigualdade?
>
>Para quem não fez a prova, o enunciado era
>
>QUESTÃO 4. Um dado é lançado três vezes e o resultado das faces é a, b
e
>c.
>Provar que P(a=c | a=b) >= P(a=c | a <> b) e que vale a igualdade se e
>somente se o dado é honesto, ou seja, a probabilidade de cada face é 1/6.
>
>Abraço, Duda.
>
>
>From: "Nicolau C. Saldanha" <[EMAIL PROTECTED]>
>> On Tue, Oct 21, 2003 at 08:58:16AM -0200, marcio.lis wrote:
>> >   Alguem poderia me informar alguma coisa sobre o q o
>> > pessoal andou fazendo na obm U informações sobre as
>> > soluções tbm seriam interessantes.Gostaria de saber se
>> > no 3 oa cardinalidade de xp=(p^2+2p+2)^2 e se no caso
>> > 2x2 ficap^2+2p+2.
>>
>> O problema 3, nível U, é de minha autoria.
>> Repetindo o enunciado, devemos contar as matrizes quadradas A
>> de tamanho 4x4 com coeficientes em Z/(p) que satisfazem A^2 = I (p >
2).
>>
>> Uma matriz A em K^(nxn), onde K é um corpo qq,
>> satisfaz A^2 = I se e somente se K^n pode ser decomposto
>> em dois subespaços U e V com interseção zero e soma K^n
>> tais que A restrito a U (resp V) é a identidade (menos a id).
>>
>> Estes dois subespaços são os autoespaços associados aos autovalores 1
>e -1.
>> Como o polinômio mínimo não tem raiz dupla, A é semisimples
>(diagonalizável).
>>
>> O importante é notar que há uma bijeção natural entre matrizes
>> satisfazendo A^2 = I e pares de subespaços U e V como acima.
>> Neste ponto dá para contar na marra ou dá para saber ou criar
>> um pouco mais de teoria.
>>
>> Na marra, você contaria para cada valor da dimensão de U.
>> Temos 2 soluções triviais com dim U = 0 e dim U = 4 (-I e I).
>> No caso dim U = 1, primeiro escolhemos U: há (p^4 - 1) geradores
>> possíveis para U mas precisamos identificar vetores que são múltiplos
>> constantes um do outro, ou seja, precisamos dividir por (p - 1)
>> para concluir que há (p^3 + p^2 + p + 1) subespaços de dimensão 1.
>> Escolha um subespaço complementar V_0 fixo qq:
>> um espaço complementar V pode ser identificado com o gráfico
>> de uma transformação linear de V_0 em U,
>> ou seja, para cada U há (p^3) espaços complementares V.
>> O caso dim U = 3 é análogo.
>> Até aqui somamos 2 p^6 + 2 p^5 + 2 p^4 + 2 p^3 + 2 e falta o caso dim
U
>=
>2.
>>
>> Para escolher um subespaço U de dim 2, vamos primeiro escolher uma base.
>> Temos (p^4 - 1) escolhas para o primeiro vetor e (p^4 - p) escolhas
>> para o segundo. Por outro lado, dado um subespaço de dim 2,
>> quantas bases ele tem? Agora temos (p^2 - 1) escolhas para o primeiro
>> vetor e (p^2 - p) escolhas para o segundo. Assim, o número de subespaços
>U
>é
>> ((p^4 - 1)(p^4 - p))/((p^2 - 1)(p^2 - p)) = p^4 + p^3 + 2p^2 + p + 1.
>> Novamente, para cada U escolha um complementar V_0 fixo qq:
>> um espaço complementar V pode ser identificado com o gráfico
>> de uma transformação linear de V_0 em U,
>> ou seja, para cada U há (p^4) espaços complementares V.
>> Ou seja, o caso dim U = 2 contribui com p^8 + p^7 + 2 p^6 + p^5 + p^4
>> e a resposta final do problema é
>>
>>  p^8 + p^7 + 4 p^6 + 3 p^5 + 3 p^4 + 2 p^3 + 2
>>
>> Para resolver o caso geral (em vez do caso 4x4),
>> ajuda muito saber contar subespaços de dimensão b de F_q^a,
>> onde q é uma potência de primo, F_q é o corpo finito de q elementos,
>> e a e b são inteiros não negativos. Este problema é tão importante
>> que a resposta tem nome, e escreve-se assim:
>>
>>( a )
>>(   )
>>( b )q
>>
>> ou seja, o símbolo de binomial com um q embaixo; eu vou escrever
>binom(a,b;q).
>> Lendo o que eu escrevi acima não é muito difícil concluir que
>>
>>  (q^a - 1)(q^(a-1) - 1)(q^(a-2) - 1)...(q - 1)
>>  binom(a,b;q) = --
>>  (q^b - 1)(q^(b-1) - 1)...(q - 1) (q^(a-b) - 1)...(q
-
>1)
>>
>> Não é muito difícil provar que isto é um polinômio em q com coeficientes
>> inteiros não negativos. A notação talvez fique menos misteriosa observando
>> que bi

[obm-l] Re: [obm-l] Grupo Abeliano

2003-10-31 Thread yurigomes

Oi Eduardo,
 
Eu acho que vc se confundiu na definição de H. Do jeito que vc colocou,
H teria n^n elementos. Eu acho que vc estava querendo dizer GxG, estou certo?
Nesse caso, H teria n^2 elementos...
Ateh mais, 
 Yuri
-- Mensagem original --

>Oi, Duda:
>
>Infelizmente, tenho que discordar. H_(n+1) soh teria n elementos se a ordem
>de g fosse n. Mas nesse caso, G seria ciclico e, portanto, abeliano.
>
>Um abraco,
>Claudio.
>
>
>on 31.10.03 00:12, Eduardo Casagrande Stabel at [EMAIL PROTECTED] wrote:
>
>> Oi Cláudio!
>> 
>> Seja G um grupo de n elementos não-abeliano.
>> Defina o grupo H = G x G x ... x G,
>> onde é o produto é tomado n vezes e estamos falando em produto
>> cartesiano. Definimos a operação de grupo em H a multiplicação das
>> coordenadas correspondentes de dois elementos quaisquer. Esta operação
>herda
>> a associatividade de G, tem elemento neutro (e, e, ..., e) e todo elemento
>> tem único inverso (g_1,g_2,...,g_n)^(-1)=(g_1^(-1),g_2^(-1),...,g_n^(-1)).
>> Como G é não-abeliano existem g, h em G tais que gh é diferente de hg,
>> portanto (g,e,e,...,e)(h,e,e,...,e) é diferente de
>> (h,e,e,...,e)(g,e,e,...,e) e H é não-abeliano. H possui exatamente n^2
>> elementos. Agora considere os subgrupos
>> 
>> H_i = { (e,e,...,e,g,e,...,e) onde o g está na i-ésima posição : para
g
>em
>> G} para 1 <= i <= n
>> e H_(n+1) = { (g,g,g,...,g) : para g em G }
>> 
>> Não é difícil de demonstrar que cada H_i é um grupo, subgrupo de H. Também
>> não é difícil de mostrar que cada um desses H_i possui exatamente n
>> elementos. Ou seja, o que está sendo pedido para demonstrar não é verdade.
>> 
>> 
>> 
>> From: "Claudio Buffara" <[EMAIL PROTECTED]>
>>> Oi, pessoal:
>>> 
>>> Me mandaram esse problema ontem e ainda nao consegui fazer:
>>> 
>>> Um grupo G de ordem n^2 tem n+1 subgrupos de ordem n tais que a
>> interseccao
>>> de quaisquer dois deles eh a trivial (ou seja, igual a {e}). Prove que
>G
>> eh
>>> abeliano.
>>> 
>>> Um abraco,
>>> Claudio.
>>> 
>>> =
>>> Instruções para entrar na lista, sair da lista e usar a lista em
>>> http://www.mat.puc-rio.br/~nicolau/olimp/obm-l.html
>>> =
>>> 
>>> 
>> 
>> =
>> Instruções para entrar na lista, sair da lista e usar a lista em
>> http://www.mat.puc-rio.br/~nicolau/olimp/obm-l.html
>> =
>> 
>
>=
>Instruções para entrar na lista, sair da lista e usar a lista em
>http://www.mat.puc-rio.br/~nicolau/olimp/obm-l.html
>=
>

[]'s, Yuri
ICQ: 64992515


--
Use o melhor sistema de busca da Internet
Radar UOL - http://www.radaruol.com.br



=
Instruções para entrar na lista, sair da lista e usar a lista em
http://www.mat.puc-rio.br/~nicolau/olimp/obm-l.html
=


[obm-l] Re: [obm-l] Polinomio divisivel por m

2003-11-23 Thread yurigomes
  Oi Claudio,
  Seja f o polinômio. Acho que uma ideia eh a seguinte: Se m= prod (i=1
até k) (p_i^(a_i)), basta verificarmos que existem n_1,..., n_k tais que

  f(n_i) = 0 (mod p_i^a_i), 
 pois tendo isso o teorema chinês dos restos ganrante que existe m satisfazendo:
  m = n_1 ( mod p_1^a_1)
   .
   . 
   .
  m = n_k ( mod p_k^a_k)
 e como a = b ( mod T ) => f(a) - f(b) ( mod T ), o resultado segue. 
  Basta então se preocupar com os primos. Isso eu acho que dah pra mostrar
por indução. Observe o seguinte:
  p=2: basta tomar x ímpar.
  p>2: vamos usar residuos quadráticos. Seja (a/p) in {-1, 0, 1} o simbolo
de Legendre. Então: 
  (13/p).(17/p)(221/p)=(13^2/p).(17^2/p)= 1.
  Logo, como temos três fatores, algum deles é igual a 1, pois caso contrario
(13/p).(17/p)(221/p) = (-1)(-1)(-1) = -1
  Suponha que tal fator seja (13/p). Então existe n tal que n^2 - 13 = 0
( mod p ). 
  Para potências maiores de p, eu vou pensar um pouquinho!
 Ateh mais, 
  Yuri

-- Mensagem original --

>Oi, pessoal:
>
>Aqui estah um problema levemente relacionado com o problema 1 da OBM nivel
>3
>desse ano (3a. fase):
>
>Prove que, para todo inteiro m (m <> 0), existe um inteiro x tal que:
>P(x) = (x^2 - 13)*(x^2 - 17)*(x^2 - 221)
>eh divisivel por m.
>
>(ou seja, pra quem conhece congruencias, P(x) == 0 (mod m) tem solucao
para
>todo m <> 0)
>
>Um abraco,
>Claudio.
>
>=
>Instruções para entrar na lista, sair da lista e usar a lista em
>http://www.mat.puc-rio.br/~nicolau/olimp/obm-l.html
>=
>

[]'s, Yuri
ICQ: 64992515


--
Use o melhor sistema de busca da Internet
Radar UOL - http://www.radaruol.com.br




=
Instruções para entrar na lista, sair da lista e usar a lista em
http://www.mat.puc-rio.br/~nicolau/olimp/obm-l.html
=


[obm-l] Re: [obm-l] Re: [obm-l] Polinomio divisivel por m

2003-11-23 Thread yurigomes
 Pronto! Soh um detalhe. O argumento que fiz abaixo mostra que existe n
tal que f(n)=0 (mod p) qdo p é diferente de 13 e 17. Para completar essa
parte, basta observar que 
 (17/13) = (4/13) = (2/13)^2 = 1.
e que pela lei de reciprocidade quadrática:
 (13/17)= (-1)^(6x8).(17/13) = 1.
 Para o caso de achar n tal que f(n)= 0 ( mod p^k), vamos usar o seguinte
lema:
  Seja f(x) in  Z[x] e A um inteiro que não  divide o coeficiente líder
de f. Se existe n tal que f(n)= 0 (mod A) e  f´(n) != 0 (mod A), então para
todo k natural existe n_k tal que 
 f(n_k)= 0 (mod A^k).
 Daí, basta ver que os m que encontramos no argumento inicial satisfazem
o lema, e o resultado segue.  

 Ateh mais, 
 Yuri
-- Mensagem original --

>  Oi Claudio,
>  Seja f o polinômio. Acho que uma ideia eh a seguinte: Se m= prod (i=1
>até k) (p_i^(a_i)), basta verificarmos que existem n_1,..., n_k tais que
>
>  f(n_i) = 0 (mod p_i^a_i), 
> pois tendo isso o teorema chinês dos restos ganrante que existe m satisfazendo:
>  m = n_1 ( mod p_1^a_1)
>   .
>   . 
>   .
>  m = n_k ( mod p_k^a_k)
> e como a = b ( mod T ) => f(a) - f(b) ( mod T ), o resultado segue. 
>  Basta então se preocupar com os primos. Isso eu acho que dah pra mostrar
>por indução. Observe o seguinte:
>  p=2: basta tomar x ímpar.
>  p>2: vamos usar residuos quadráticos. Seja (a/p) in {-1, 0, 1} o simbolo
>de Legendre. Então: 
>  (13/p).(17/p)(221/p)=(13^2/p).(17^2/p)= 1.
>  Logo, como temos três fatores, algum deles é igual a 1, pois caso contrario
>(13/p).(17/p)(221/p) = (-1)(-1)(-1) = -1
>  Suponha que tal fator seja (13/p). Então existe n tal que n^2 - 13 =
0
>( mod p ). 
>  Para potências maiores de p, eu vou pensar um pouquinho!
> Ateh mais, 
>  Yuri
>
>-- Mensagem original --
>
>>Oi, pessoal:
>>
>>Aqui estah um problema levemente relacionado com o problema 1 da OBM nivel
>>3
>>desse ano (3a. fase):
>>
>>Prove que, para todo inteiro m (m <> 0), existe um inteiro x tal que:
>>P(x) = (x^2 - 13)*(x^2 - 17)*(x^2 - 221)
>>eh divisivel por m.
>>
>>(ou seja, pra quem conhece congruencias, P(x) == 0 (mod m) tem solucao
>para
>>todo m <> 0)
>>
>>Um abraco,
>>Claudio.
>>
>>=
>>Instruções para entrar na lista, sair da lista e usar a lista em
>>http://www.mat.puc-rio.br/~nicolau/olimp/obm-l.html
>>=
>>
>
>[]'s, Yuri
>ICQ: 64992515
>
>
>--
>Use o melhor sistema de busca da Internet
>Radar UOL - http://www.radaruol.com.br
>
>
>
>
>=
>Instruções para entrar na lista, sair da lista e usar a lista em
>http://www.mat.puc-rio.br/~nicolau/olimp/obm-l.html
>=
>

[]'s, Yuri
ICQ: 64992515


--
Use o melhor sistema de busca da Internet
Radar UOL - http://www.radaruol.com.br




=
Instruções para entrar na lista, sair da lista e usar a lista em
http://www.mat.puc-rio.br/~nicolau/olimp/obm-l.html
=


[obm-l] Re: [obm-l] Teoria dos números....

2004-02-03 Thread yurigomes
 Oi Crom, 

 Tem algo errado nessa fração. Tome n=5. Então, se não interpretei errado,
a fração vale:
4.(4! + 1)/(5.7) = 4.25/(5.7) = 20/7, que não é inteiro...

 Ateh mais, 
 Yuri

 
-- Mensagem original --

>1)Seja n>=2 um número ineiro. Prove que n e n+2 são ambos primos se e somente
>
>se 4((n-1)! + 1)/n(n+2) é inteiro.
> Desde já agradeço a quem fizer.
>   Crom
>

[]'s, Yuri
ICQ: 64992515


--
Use o melhor sistema de busca da Internet
Radar UOL - http://www.radaruol.com.br




=
Instruções para entrar na lista, sair da lista e usar a lista em
http://www.mat.puc-rio.br/~nicolau/olimp/obm-l.html
=


[obm-l] Re: [obm-l] Mais Ramsey

2004-03-04 Thread yurigomes


> Prove que se pintarmos cada aresta de um grafo completo de
>ordem 6 com uma dentre duas cores, este grafo conterah um subgrafo
>monocromatico de ordem 3 (um triangulo, por assim dizer).

Na verdade, é possível mostrar que existem DOIS triângulos monocromáticos!

Até mais,
 
Yuri

[]'s, Yuri
ICQ: 64992515


--
Use o melhor sistema de busca da Internet
Radar UOL - http://www.radaruol.com.br




=
Instruções para entrar na lista, sair da lista e usar a lista em
http://www.mat.puc-rio.br/~nicolau/olimp/obm-l.html
=


[obm-l] Re: [obm-l] OLIMPÍADA CEARENSE!

2004-03-05 Thread yurigomes
 Eu fiz essa prova! Eu acho

  Observe o seguinte: os números 2, 4, 8, 16 e 32 devem estar em caixas
distintas, pois senão a condição do mdc não seria satisfeita. Então temos
um total de caixas maior ou igual a 5. Agora basta mostrar um exemplo com
5 caixas. Acho que colocando os números 2^i, 2^i + 1, ..., 2^(i+1) - 1 na
caixa i dá certo. Quer dizer:
Caixa 1 -> 2, 3;
Caixa 2 -> 4, 5, 6, 7;
Caixa 3 -> 8 a 15;
Caixa 4 -> 16 a 31;
Caixa 5 -> 32 a 51.
 De fato, se a, b estão na mesma caixa, por exemplo na 3,  e d=mdc(a, b)
, então d|(b-a) < 8, de modo que d não está nessa caixa. Em geral, se a,
b pertencem à caixa i, então d <= b-a < 2^i => d não pertence à caixa i.


Até mais,
 
 Yuri
-- Mensagem original --

>Olá! Meus Amigos! Sou muito grato as elucidações e valiosas informações

>enviadas, pois não suspeitava da complexidade do probleminha clássico que
>
>enviei recentemente à lista. Também gostei do improviso da receita de biscoito
>
>e aproveitando o clima amigável, gostaria que atendessem ao pedido da Renata
>
>sobre a resolução do tal "problema esquisito" proposto na Olimpíada Cearense.
>
>Agora, sem querer abusar da boa vontade dos nobres colegas, vejam outro
que
>
>caiu em nossa singela Olimpíada. OBRIGADO!
>
>Cinquenta bolas, numeradas de 2 a 51, devem ser colocadas em caixas, de
modo
>
>que o máximo divisor comum dos números de duas bolas quaisquer de uma caixa
>não 
>seja o número correspondente a uma bola desta caixa. Encontre o número
mínimo
>
>de caixas necessárias para guardar todas as bolas. Justifique sua resposta.
>
>Bom Final de Semana!
>
>
>
>
>WebMail UNIFOR - http://www.unifor.br
>=
>Instruções para entrar na lista, sair da lista e usar a lista em
>http://www.mat.puc-rio.br/~nicolau/olimp/obm-l.html
>=
>

[]'s, Yuri
ICQ: 64992515


--
Use o melhor sistema de busca da Internet
Radar UOL - http://www.radaruol.com.br




=
Instruções para entrar na lista, sair da lista e usar a lista em
http://www.mat.puc-rio.br/~nicolau/olimp/obm-l.html
=